polynomials - pedagogická fakulta mu€¦ · web viewfrom the algebraic point of view it is...

81
Polynom ials Text for Students of Mathematics Teaching Irena Budínová

Upload: others

Post on 24-Aug-2020

1 views

Category:

Documents


0 download

TRANSCRIPT

Page 1: Polynomials - Pedagogická fakulta MU€¦ · Web viewFrom the algebraic point of view it is especially interesting that the result of adding and multiplying polynomials is again

Polynomials

Text for Students of Mathematics Teaching

Irena Budínová

Page 2: Polynomials - Pedagogická fakulta MU€¦ · Web viewFrom the algebraic point of view it is especially interesting that the result of adding and multiplying polynomials is again

Masaryk University

Faculty of Education

POLYNOMIALS

TEXT FOR STUDENTS OF MATHEMATICS TEACHING

MGR. IRENA BUDÍNOVÁ, PH.D.

2013

Page 3: Polynomials - Pedagogická fakulta MU€¦ · Web viewFrom the algebraic point of view it is especially interesting that the result of adding and multiplying polynomials is again

1 Table of Contents2 Polynomials...............................................................................................................................5

Chapter Objectives........................................................................................................................5

Time to Study................................................................................................................................5

Study Guide...................................................................................................................................5

2.1 Algebraic Definition of Polynomial....................................................................................6

2.2 Polynomial Division...........................................................................................................8

2.3 Horner’s Method.............................................................................................................10

2.4 Application of Horner Scheme for Taylor Expansion.......................................................12

2.5 Divisibility of Polynomials................................................................................................18

Exercises for Check......................................................................................................................26

3 Algebraic Equations.................................................................................................................27

Chapter objectives.......................................................................................................................27

Time to Study..............................................................................................................................27

Study Guide.................................................................................................................................27

3.1 Roots of Polynomial.........................................................................................................28

3.2 Polynomial Derivative......................................................................................................31

3.3 Polynomials with Integer Coefficients.............................................................................33

3.4 Vieta’s Formulas..............................................................................................................36

3.5 Linear Equation................................................................................................................39

3.6 Quadratic Equation..........................................................................................................39

3.7 Cubic Equation.................................................................................................................40

3.8 Fourth Degree Equation..................................................................................................41

3.9 Higher Degree Equations.................................................................................................41

3.10 Selected Types of Algebraic Equations............................................................................42

Exercises for Check..........................................................................................................................47

4 Multivariate Polynomials.........................................................................................................48

Chapter Objectives......................................................................................................................48

Time to Study..............................................................................................................................48

Study Guide.................................................................................................................................48

4.1 Elementary Symmetric Polynomials................................................................................51

Exercises for Check......................................................................................................................57

References.......................................................................................................................................58

3

Page 4: Polynomials - Pedagogická fakulta MU€¦ · Web viewFrom the algebraic point of view it is especially interesting that the result of adding and multiplying polynomials is again
Page 5: Polynomials - Pedagogická fakulta MU€¦ · Web viewFrom the algebraic point of view it is especially interesting that the result of adding and multiplying polynomials is again

2 Polynomials

Chapter Objectives

Having studied this chapter you will be able to:

define polynomial divide polynomials perform Taylor series expansion find the greatest common divisor of polynomials find roots of polynomials use Vieta’s formulas for searching for roots of polynomials

Time to Study

Circa 3 hours

Study Guide

The notion of polynomial must be already known to you from a high school. A polynomial (also known as a multinomial) was seen as a mathematical expression, which is expressed as a sum of several monomials. A monomial consists of several parts – a coefficient (which can be any real number) and one or more variables with a positive integer exponent.

In a mathematical analysis course you have encountered a polynomial as a special case of a (real) function (of a single real variable), id est as a mapping from the set of real numbers into the set of real numbers. These functions are very pleasant to work with due to their properties: their domain of definition is always the whole set of real numbers, they are continuous on this whole set and have derivations of any order which are polynomials again, etc.

At elementary school you have learned to add polynomials and to multiply them. At high school your skills have been extended by dividing polynomials.

From the algebraic point of view it is especially interesting that the result of adding and multiplying polynomials is again a polynomial. Thence polynomials form an algebraic structure with two (binary) operations.

5

Page 6: Polynomials - Pedagogická fakulta MU€¦ · Web viewFrom the algebraic point of view it is especially interesting that the result of adding and multiplying polynomials is again

Let us remind the form in which you are used to writing polynomials. Any polynomial can be transformed into the form

an xn+an−1 xn−1+…+a1 x+a0 ,

where n∈N ,a0 , a1 ,…, an∈R.

In this text we will extend your knowledge in the field of polynomials by important concept definitions and by other possibilities of polynomials manipulation.

We will use concepts and knowledge you have already encountered in previous algebra courses. Proofs of theorems will be included only in case the proof is not overly demanding and there can be a certainty that the student will be able to follow the line of the proof. End of a proof will be marked by a square.

When constructing polynomials we have basically two ways we can proceed with. We can regard them as functions, which you have come across with in mathematical analysis. In this field a polynomial is perceived as a function of a single variable x, which can be in the whole domain written as an xn+an−1 xn−1+…+a1 x+a0, where a0 , a1 ,…,an are constants. This definition states that every polynomial can be expressed as a finite sum of monomials in the form ak xk, where the variable is exponentiated to a nonnegative integer.

Yet we can also look at them from the point of view that a polynomial is defined by its coefficients and that the polynomials equality, addition and multiplication can be described by equalities and operations of their coefficients. We will use this second approach and will base the study on the algebraic definition of polynomial.

2.1 Algebraic Definition of Polynomial

Definition 1 By a polynomial over a field T1 we mean a sequence ¿¿) of elements of the field T, in which there is at most a finite count of nonzero elements. The set of all polynomials over T is denoted T[x].The polynomial o=(0,0 , …,0) is called a zero polynomial.

1 By a numerical field we mean an ordered triple ¿¿, where T is a subset of the set of complex

numbers C, such that 0∈T , 1∈T and there applies:

(∀ x , y∈T )(x+ y∈T∧x . y∈T )(it is closed for both addition and multiplication),

(∀ x∈T )(−1) . x∈T (it is closed for inverse elements),

(∀ x∈T )( x≠0⇒ 1x∈T )

(it is closed for inverse values of nonzero elements).

Page 7: Polynomials - Pedagogická fakulta MU€¦ · Web viewFrom the algebraic point of view it is especially interesting that the result of adding and multiplying polynomials is again

Note: For instance a polynomial 5 x7+13 x6−2 x3+2 can be identified with the sequence (2, 0, 0,-2 ,0 ,0 ,13 ,5).

Definition 2 Let us define over the set T[x] that for f =(a0 , a1, …), g= (b0 , b1 ,… )∈T [ x ]:

1) f =g ∀ k∈N0 :ak=bk

2) f +g=(a0+b0 , a1+b1 ,…, ak+bk , …), f +g∈T [ x ]3) f ∙ g=(c0 , c1, c2 ,…), f ∙ g∈T [ x ], where

c0=a0 b0 , c1=a0 b1+a1b0 ,c2=a0 b2+a1 b1+a2b0, etc.

Note: E.g. the polynomials f =2 x3+x2−x+3 and g=3 x2+2 x+1 are multiplied in this way:

f ∙ g=2 x3 ∙ 3 x2+2 x3 ∙2 x+2 x3 ∙1+ x2 ∙3 x2+x2 ∙ 2x+x2 ∙ 1−x ∙3 x2−x ∙ 2x−x ∙ 1+3∙3 x2+3 ∙2 x+3 ∙1=6 x5+4 x4+2 x3+3x4+2x3+x2−3 x3−2x2−x+9 x2+6 x+3=6 x5+7 x4+x3+8 x2+5 x+3 ,hence c0=3=3 ∙1 , c1=5=3 ∙2−1∙ 1, etc. The polynomial f ∙ g can be written in the form (3 , 5 , 8 ,1 , 7 ,6).

Theorem 1 (T [x ] ,+ ,.)is an integral domain2 for each field T .

Proof:

Addition of polynomials is commutative, i.e. f +g=g+ f .

Addition of polynomials is associative, i.e. f +(g+h )= (f +g )+h.

The zero element is the zero polynomial.

The inverse element for a polynomial f =(a0 , a1 , …) is a polynomial − f =(−a0 ,−a1 , …).

Multiplication of polynomials is commutative. Multiplication of polynomials is associative. The identity element is the polynomial i=(1,0,0 , …,0). Zero divisors: it is obvious that if polynomials f and g are nonzero then their

product is nonzero too.

Note: Following is the demonstration how a polynomial represented using a sequence can be turned into the expression familiar from high school.

Let the polynomial f =(a ,0,0 , …) identify with an element a∈T ,

(a , 0,0 , …)≡ a.

Next let the polynomial (0,1,0,0 ,…) be denoted as x, (0,0,1,0 …) as x2 etc. Then there applies

2 Let us remind that an integral domain is a commutative ring that contains the identity element and where the zero element does not have any non-trivial divisors.

7

Page 8: Polynomials - Pedagogická fakulta MU€¦ · Web viewFrom the algebraic point of view it is especially interesting that the result of adding and multiplying polynomials is again

(a0 , a2 ,…, ak )=( a0 ,0,0 , … )+ (0 , a1 , 0 , …)+…+( 0 , …,ak ,0 , … )=¿

¿a0 (1,0 , … )+a1 (0,1,0 , … )+a2 (0,0,1,0 ,… )+…+ak (0,0 , …, 1,0 , … )=¿

¿a0+a1 x+a2 x2+…+ak xk.

Definition 3 Let f ∈T [x ]. Then the degree of a polynomial (dg f ) is defined in this way: if f ≠ o, f =a0+a1 x+…+an xn, an≠ 0, then it is defined

dg f =n.

Theorem 2 For degrees of polynomials f , g∈T [ x ] there applies:1) dg (f +g)≤ max {dg f , st g },2) dg ( fg )=dg f +dg g

Proof: The proof of the first part is obvious, thus part 2: f =a0+a1 x+…+ak xk, g=b0+b1 x+…+b l x

l, dg f =k, dg g=l, fg=a0 b0+a0 b1 x+…+ak bl xk+l and it is apparent that the product ak b l is nonzero. Hence the degree of the polynomial fg is k+l.

Note: Let us consider the polynomials f =x3+2 x2+x+1, g=−x3+x2−1, dg f =3 , dg g=3, a polynomial f +g=(1−1 ) x3+(2+1 ) x2+(1+0 ) x+1−1=3 x2+x, dg (f +g)=2, f ∙ g=−x6−x5+x4−x3−x2−x−1, dg ( f ∙ g )=6.

2.2 Polynomial Division

For polynomial division the knowledge of integer division will be applied. Surely, you can remember the algorithm to divide for instance a five-digit number by a two-digit one. The process of polynomial division is very similar.

Just as when dividing two integers over the integer domain the result can be either without or with remainder, the result of division of two polynomials can likewise be a polynomial without a remainder, or so-called partial quotient and a remainder.

Note: Let us remind the theorem regarding integer division with a remainder: For each pair of integers a , b∈Z, where b ≠ 0, there exists exactly one pair of integers q ,r∈Z such that there applies:

1. a=bq+r,2. 0 ≤ r<¿b.

For instance, we can consider numbers a=51 , b=5. We know that 51 :5=10+ 15 ,

thus the partial quotient is 10 and the remainder 1. This fact can be expressed as51=5 ∙ 10+1.

Page 9: Polynomials - Pedagogická fakulta MU€¦ · Web viewFrom the algebraic point of view it is especially interesting that the result of adding and multiplying polynomials is again

The form 51=5 ∙9+6 could also be chosen to be used, but according to the previous theorem, the remainder of division by five can attain values 0, 1, 2, 3 or 4.

The theorem regarding polynomial division is analogical. Before it is introduced let us remind the algorithm for dividing two polynomials, well known from high school.

Note (Algorithm for Dividing Two Polynomials): Before dividing two polynomials f and g, it is first necessary to check whether both of them are ordered by their power x descending. We divide the first term of the polynomial f by the first term of the polynomial g, with the result of φ1. Then we multiply the polynomial φ1 with the polynomial g and subtract the resulting polynomial g φ1 from the polynomial f . The polynomial f −g ∙φ1 will be denoted as f 1. If dg f 1≥ st g we continue with dividing. We then divide the first term of the polynomial f 1 by the first term of the polynomial g again and proceed analogically. The succession of steps will be illustrated on the following example.

Example 1 Divide polynomials f , g, when f =2 x4−3 x3+x2−5 x+2, g= x2−3 x+2.

Solution: φ1φ2φ3

↓ ↓↓

(2 x4−3 x3+x2−5 x+2 ) : ( x2−3 x+2 )=2x2+3 x+6+ rg

g .φ1: 2x4−6 x3+4 x2

f 1=f −g . φ1 :3 x3−3 x2−5 x+2

g . φ2: 3 x3−9x2+6 x

f 2=f −g . φ1−g . φ2 :6 x2−11 x+2

g . φ3 :6 x2−18x+12

r :7 x−10

Hence the result of division is a partial quotient q ( x )=2x2+3 x+6 and a remainder r (x )=7 x−10.

Theorem 3 (Polynomial Long Division) For each pair of polynomials f , g∈T [ x ], where g ≠ 0, there exists exactly one pair of polynomials q ,r∈T [ x ] for which there applies:

1) f =g ∙ q+r ,

9

Page 10: Polynomials - Pedagogická fakulta MU€¦ · Web viewFrom the algebraic point of view it is especially interesting that the result of adding and multiplying polynomials is again

2) dg r<dg g.

Proof: Firstly, the existence of polynomials q ,r will be proved:a) When dg f <st g, then q=0 , r=f .b) When dg f ≥ st g, then we follow the division algorithm.

f −g ∙φ1=f 1 , dg f 1<dg f . If dg f 1≥ dg g, thenf 1−g ∙φ2= f 2 , dg f 2<dg f ,⋮ f k−1−g ∙ φk=f k , dg f k<dg g.

By adding these equations together we get f −g ∙(φ¿¿1+…+φk)=f k ¿, and by denoting (φ ¿¿1+…+φk)=q¿ and f k=r , we come to f =g ∙ q+r. By this we have proven the first part of the theorem.Secondly, there will come the proof that there exists exactly one such pair of polynomials. The proof will be performed by contradiction, let us therefore assume that there could exist two different pairs of these polynomials:

f =g ∙q+r , dg r<dg g , f =g ∙ q+r , dg r<dg g.

By subtraction we get 0=g ∙ (q−q )+(r−r ) r−r=g(q−q). It is then apparent that dg g>dg (r−r )=dg g+dg(q−q) and since dg g=dg g, there must apply q−q=0, thus q=q and from the validity of the equation r−r=g (q−q) there must also apply r=r. We have proven that it is the same pair of polynomials, which is contradictory with the initial assumption. By this the proof has been performed.

Example 2 Solve individually: Compute q ( x ) , r (x), for f ( x )=x5− x3+3 x−5,

a) g ( x )=x2+7,

b) g ( x )=x−2.

Result: a) q ( x )=x3−8 x , r ( x )=59 x−5, b) q ( x )=x4+2 x3+3 x2+6 x+15 , r (x )=25.

2.3 Horner’s Method

Let us say that our task is to find the value of the polynomial 5 x4−4 x3+7 x2−x+1 for x=4. We can certainly substitute 4 for x, but let us assume we are indolent to exponentiate 44, and therefore we need some other approach with only simpler expressions involved.

The procedure will be as follows: Factoring out x3 from the first two polynomial terms, we get (5 x−4)x3. Then, from this newly formed expression and the third term of the polynomial, we factor out x2, resulting into [ (5 x−4 ) x+7 ] x2. From this expression and

Page 11: Polynomials - Pedagogická fakulta MU€¦ · Web viewFrom the algebraic point of view it is especially interesting that the result of adding and multiplying polynomials is again

the fourth term, when factoring out x, we get {[ (5 x−4 ) x+7 ] x+1} x. Altogether the original polynomial will gain the form

{[ (5 x−4 ) x+7 ] x+1} x+1

and by substituting x with 4 we get {[ (16 ) 4+7 ] 4+1 } 4+1=1141.

The previous example can be generalised. When dg f =n≥ 1 , f =a0 xn+a1 xn−1+…+an ,c∈T , g=x−c then there exists a polynomial q ,dg q=n−1 , q=b0 xn−1+b1 xn−2+…+bn−1 , r=bn and there applies

a0 xn+a1 xn−1+…+an=( x−c ) (b0 xn−1+…+bn−1 )+bn .

This situation can be expressed in a table called a Horner scheme:

a0 a1 … an−1 an

c b0 b1 … bn−1 bn

When expanding the right side of the previous equation (you can perform by yourselves to ascertain the validity of the statement), we get:

a0=b0

a1=b1−cb0

a2=b2−cb1

an=bn−c bn−1

We are interested in the number bn, which is the value of the given polynomial at the point c. Therefore we transform the previous sequence into

b0=a0

b1=c b0+a1

b2=c b1+a2

bn=c bn−1+an .

Example 3 Determine the value of a polynomial x4−3 x3+2 x2−2 x−16 for x=5.

Solution: The value of the polynomial at the given point will be found by using Horner’s method. We will proceed according to the fore-mentioned sequence

b0=a0

11

Page 12: Polynomials - Pedagogická fakulta MU€¦ · Web viewFrom the algebraic point of view it is especially interesting that the result of adding and multiplying polynomials is again

b1=c b0+a1

b2=c b1+a2

bn=c bn−1+an .

In our case, we place the polynomial coefficients into the first row of the table and the value x=5 into the first column. Then we write the first coefficient b0=a0=1 into the second row and calculate on: 5 ∙1−3=2 ;5 ∙2+2=12;5 ∙12−2=58 ;5 ∙ 58+16=274.

1 -3 2 -2 16

5 1 2 12 58 274

That means we have found out that the value of the polynomial at the point 5 is 274. Yet the result of the table can be interpreted in another way, too. When dividing the polynomial x4−3x3+2 x2−2 x−16 by the polynomial x−5, we get the partial quotient q ( x )=x3+2 x2+12 x+58 together with the remainder r ( x )=274. This fact can be verified by dividing them. The result can be expressed as x4−3x3+2 x2−2 x−16=(x¿¿3+2x2+12 x+58) (x−5 )+274 ¿.

Hence Horner’s method can be used for simple division of a polynomial by a linear polynomial, as demonstrated in the following example.

Example 4 Divide the polynomials from the example 2 b) using Horner’s method.

Solution: 1 0 -1 0 3 -5

2 1 2 3 6 15 25

The result is therefore q ( x )=x4+2 x3+3 x2+6 x+15 , r (x )=25.

Example 5 Solve individually: Determine the value of the polynomial f ( x )=x5−4 x3+2 x2−7 at the point x=6. Write in the form f ( x )=q ( x ) g ( x )+r (x).

Result: f (6 )=6977 , x5−4 x3+2 x2−7=( x4+6 x3+32 x2+194 x+1164 ) ( x−6 )+6977

2.4 Application of Horner Scheme for Taylor Expansion of Polynomial

You certainly can recall Taylor series that could be used in mathematical analysis to replace some functions in the neighbourhood of a given point. Let us remind Taylor’s

Page 13: Polynomials - Pedagogická fakulta MU€¦ · Web viewFrom the algebraic point of view it is especially interesting that the result of adding and multiplying polynomials is again

theorem, which states that a function f (x), having in the neighbourhood of a point a derivations up to an order of (n+1), can be in the neighbourhood of the point a expressed as

f ( x )=f (a )+ f ' (a )1!

( x−a )+ f ' ' (a )2!

( x−a )2+…+f (n) (a )

n !( x−a )n+R ,

where f ( a )+ f ' (a )1 !

( x−a )+ f ' ' (a )2 !

( x−a )2+…+ f (n )(a )n !

( x−a )n is the Taylor series and R is the

Taylor series remainder. This way the approximate value of the given function in the point a can be found.

In case the function f (x) is a polynomial, there exists a much easier way of finding Taylor series, which is by the application of Horner’s method.

Definition 4 Let f ∈T [ x ] , f ( x )=a0 xn+a1 xn−1+…+an , st f =n , c , c1 ,…, cn∈T . Then if f =d0+d1 ( x−c )+d2 ¿

where d0 , …, dn∈T , then the expression on the right side is called a Taylor series (Taylor expansion) centred at c.

Theorem 4 For each c∈T , f ∈T [ x ] there exists a Taylor series of a polynomial f centred at c.

Proof: When dg f =0, then f =d0. When st f ≥ 1, then after dividing the polynomial f by the expression (x−c) we get

f =( x−c ) f 1+d0 , dg f 1=n−1

f 1=( x−c ) f 2+d1 , dg f 2=n−2

f n−1=( x−c ) f n+dn−1 , dg f n=n−n=0

which means that f n is a constant polynomial. Now when we leave the polynomial f as it is, then multiply the polynomial f 1 by the expression (x−c), the polynomial f 2 by the expression (x−c)2, etc., up to multiplying the polynomial f n by the expression (x−c)n and when we sum up all the resulting polynomials, we come to the Taylor series centred at c, i.e.

f =d0+d1 ( x−c )+d2 ¿

Example 6 Given the polynomial f ( x )=a0 xn+a1 xn−1+…+an, apply the Horner’s method to find a Taylor series centred at c.

Solution: We are to rearrange the polynomial f in the following way: f =( x−c ) f 1+d0=( x−c ) [ ( x−c ) f 2+d1 ]+d0=…. The coefficients of the polynomial will be

13

Page 14: Polynomials - Pedagogická fakulta MU€¦ · Web viewFrom the algebraic point of view it is especially interesting that the result of adding and multiplying polynomials is again

written into the first line, and the centre c into the first column. In the first step we proceed like we did in Horner scheme higher above. The last number in the second row is d0. In the second step we do not work with coefficients of the original polynomial anymore, but with the coefficients of the polynomial f 1, instead, i.e. with the numbers from the second row. The last number of the row is left out, processing only up to d1. In this way we advance onwards. The table has the following form:

a0 a1 … an

c a0 … d0

c a0 d1

c a0 d2

c ⋮

c dn

The numbers d0 ,…, dn are the sought Taylor series coefficients.

Example 7 For the polynomial f ( x )=x5 find a Taylor series centred at c=1.

Solution: 1 0 0 0 0 0

1 1 1 1 1 1 11 1 2 3 4 51 1 3 6 101 1 4 1

01 1 51 1

Therefore the result is

x5=1+5 ( x−1 )+10(x−1)2+10(x−1)3+5(x−1)4+(x−1)5

Note: There is a noticeable similarity of Taylor series coefficients from the previous example with the Pascal’s triangle coefficients:

11 1

1 2 11 3 3 1

Page 15: Polynomials - Pedagogická fakulta MU€¦ · Web viewFrom the algebraic point of view it is especially interesting that the result of adding and multiplying polynomials is again

1 4 6 4 11 5 10 10 5 1

1 6 15 20 15 6 1

Examine for yourselves whether for polynomials x3 or x6 their coefficients of a Taylor series centred at c=1 are also equal to the Pascal’s triangle coefficients. Examine further what do Taylor series with different centres look like, for instance for c=−1, c=2.

Example 8 Solve individually: For the polynomial f ( x )=x4−8 x3+24 x2−50 x+90 find out a Taylor series centred at c=2.

Result: x4−8 x3+24 x2−50 x+90=38−18 ( x−2 )+(x−2)4.

Definition 5 Let f ∈T [ x ] , f ( x )=a0 xn+a1 xn−1+…+an , dg f =n , c , c1 , …, cn∈T . If f =d0+d1 ( x−c1 )+d2 ( x−c1 ) ( x−c2 ) …+dn ( x−c1 ) ( x−c2 ) … ( x−cn ) ,

then the right side is called a Gaussian interpolation polynomial.

Theorem 5 Let f ∈T [ x ] , dg f =n≥ 1, c1 , c2 ,…,cn∈T . Then there exist numbers d1 , d2 ,…, dn∈T such, that there applies f =d0+d1 ( x−c1 )+d2 ( x−c1 ) ( x−c2 )+…+dn ( x−c1 ) ( x−c2 ) …( x−cn). The polynomial f can therefore be expressed in the form of a Gaussian interpolation polynomial with chosen numbers c1 , c2 ,…, cn.

Proof: When dividing the polynomial f by the expression ( x−c1 ) , then by the expression ( x−c2 ), etc., we get:

f =( x−c1 ) f 1+d0 .1

f 1=( x−c2 ) f 2+d1 .(x−c1)

f n−1=( x−cn ) f n+dn−1 . ( x−c1 ) ( x−c2 ) …(x−cn−1)

f n=dn . ( x−c1 ) ( x−c2 ) …(x−cn)

When we multiply the resulting polynomials back again with the expressions in the right column and sum them up, we come to f =d0+d1 ( x−c1 )+d2 ( x−c1 ) ( x−c2 )+…+dn ( x−c1 ) ( x−c2 ) …( x−cn).

Example 9 Express the polynomial f =2 x5−3 x4−5 x3+2x2+7 x−9 in the form of Gaussian interpolation polynomial, given c1=1 , c2=2 , c3=−3 , c4=2 , c5=−1.

15

Page 16: Polynomials - Pedagogická fakulta MU€¦ · Web viewFrom the algebraic point of view it is especially interesting that the result of adding and multiplying polynomials is again

Solution: We will use similar procedure as when seeking a Taylor series by application of Horner’s method. Into the left column we will put values c1 , …,c5 successively.

2 -3 -5 2 7 -9

1 2 -1 -6 -4 3 -6

2 2 3 0 -4 -5

-3 2 -3 9 -31

2 2 1 11

-1 2 -1

2

The polynomial is then of the form f =−6−5 ( x−1 )−31 ( x−1 ) ( x−2 )+11 ( x−1 ) (x−2 ) ( x+3 )−¿( x−1 ) ( x−2 ) ( x+3 ) ( x−2 )+2 ( x−1 ) ( x−2 ) (x+3 ) ( x−2 )(x+1).

Note: The Gaussian interpolation polynomial can be used to find a polynomial of order n specified by values at n+1 points, as will be demonstrated in the next example.

Let us merely remind that two points uniquely specify a straight line, three points specify a parabola, and so forth. For determining of a first-degree polynomial we therefore need values of two points, for a polynomial of the second degree we need values of three points, and generally for finding out of a polynomial of degree (n+1) there are n points needed.

Example 10 Find a polynomial f ∈ R [ x ] for which there applies f (−2 )=3 , f (−1 )=1 , f (2 )=−2 , f (3 )=5.

Solution: Since the sought polynomial will be apparently of the third degree, we can express it in the form of the Gaussian interpolation polynomial, where we will choose to substitute c1 , c2 , c3 with values of independent variable −2 ;−1 ;2,3 resulting to f =d0+d1 ( x+2 )+d2 ( x+2 ) (x+1 )+d3 ( x+2 ) ( x+1 )(x−2).

In this form of the polynomial we substitute x with c1=−2 and as according to the assignment there applies f (−2 )=3, we can replace f with 3, thus 3=d0+d1 (−2+2 )+d2 (−2+2 ) (−2+1 )+d3 (−2+2 ) (−2+1 )(−2−2), and we find out that d0=3.

Similarly continuing for c2=−1, f (−1 )=1, after rearrangement we get 1=3+d1(−1+2), and then d1=−2.

3 The symbol R [ x ] stands for a set of all polynomials over the field of real numbers.

Page 17: Polynomials - Pedagogická fakulta MU€¦ · Web viewFrom the algebraic point of view it is especially interesting that the result of adding and multiplying polynomials is again

Next we set x to c3=2 , f (2 )=−2, which results into d2=14 . Lastly c4=3 , f (3 )=5, the

result is d3=720 .

The form of the polynomial is hence

f =3−2 ( x+2 )+ 14

( x+2 ) ( x+1 )+ 720

( x+2 ) (x+1 )(x−2). After expansion we get

f = 720

x3+35

x2−5320

x−1910

.

Now you can check by yourselves whether the values from the assignment comply with the resulting polynomial.

Example 11 Solve individually: By an application of the Gaussian interpolation polynomial, determine f ∈ R [ x ] such that there applies

a) f (2 )=2 , f (−2 )=−4,

b) f (−1 )=92

, f (0 )=4 , f (2 )=12.

Result: a) f =32

x−1, b) f =3 x2−12

x+1

Definition 6 Let f ∈T [ x ] , f ( x )=a0 xn+a1 xn−1+…+an , dg f =n , c , c1 , …, cn∈T . Considering f =L1 (x ) y1+ L2 ( x ) y2+…+ Ln ( x ) yn, where y1 , …, yn∈T ,

L1 ( x )=( x−c2) ( x−c3 )… (x−cn)

( c1−c2) ( c1−c3 ) …(c1−cn),

Ln ( x )=( x−c1 ) ( x−c2 ) …(x−cn−1)

( cn−c1 ) (cn−c2 ) …(cn−cn−1).

If c i ≠ c j for i≠ j, then the right side is called a Lagrange (interpolation) polynomial.

Note: Lagrange polynomial can be, similarly as Gaussian interpolation polynomial, applied to find a polynomial passing through given points on a certain interval.

Example 12 Use the Lagrange polynomial to find the polynomial that passes through the points [−1 ;9 ] , [ 1 ;1 ] , [ 2;6 ].

Solution: The form of the polynomial will be f =9 L1 (x )+L2 ( x )+6 L3(x ), where

L1 ( x )= (x−1 ) ( x−2 )(−1−1 ) (−1−2 )

=16(x2−3 x+2),

17

Page 18: Polynomials - Pedagogická fakulta MU€¦ · Web viewFrom the algebraic point of view it is especially interesting that the result of adding and multiplying polynomials is again

L2 (x )= ( x+1 ) (x−2 )(1+1 ) (1−2 )

=−12

(x2−x−2) ,

L3 (x )= ( x+1 ) (x−1 )(2+1 ) (2−1 )

=13

( x2−1 ) .

After substitution we get f =96

( x2−3 x+2 )−12

( x2−x−2 )+ 63

( x2−1 )=3 x2−4 x+2. You can

check by yourselves that the polynomial conforms to the assignment requirements.

Example 13 Solve individually: Use the Lagrange polynomial to solve Example 11.

Example 14 Solve individually: By an application of the Lagrange polynomial find a polynomial that passes through the points [−1 ;0 ] , [ 0 ;1 ] , [ 1 ;3 ] , [ 2;−2 ].

Result: f =−43

x3+12

x2+ 176

x+1

2.5 Divisibility of Polynomials

Definition 7 Let f , g∈T [ x ]. We say that a polynomial g divides a polynomial f if and only if there exists any such polynomial h∈T [ x ] for which there applies f =g .h. Then we write gf .

Theorem 6 (Basic Properties of Divisibility of Polynomials)

Let f , g∈T [ x ], f ≠ 0 , g≠0. Then there applies:

1) f f

2) For all u , v  ∈ T [ x ] and all f 1 , f 2 ∈ T [ x ] there applies: If g|f 1 and at the same time g|f 2, then also g|( f 1u+ f 2 v ).

3) Let h∈T [ x ], then there applies: If h|g and also g|f , then h|f as well.

4) g|f dg g ≤ dg f

5) g|f and f|g at the same time if and only if there exists such nonzero element c∈T , for which there applies g=cf .

Proof:

Re 1) f =1. f =f .1

Re 2) g|f 1∃h1∈T (x ) : f 1=g .h1, g|f 2∃ h2∈T (x ) : f 2=g . h2. Then f 1 . u+ f 2 . v=g .h1. u+g . h2 . v=g . ( h1u+h2 v ), upon denoting h1 u+h2 v=h, h∈T [ x ], we get f 1. u+ f 2. v=h.

Page 19: Polynomials - Pedagogická fakulta MU€¦ · Web viewFrom the algebraic point of view it is especially interesting that the result of adding and multiplying polynomials is again

Re 3) If h|g then g=hφ1 and similarly if g|f , then f =g φ2. Therefore f =g φ2=(h φ1 ) φ2=h(φ ¿¿1 φ¿¿2)¿ ¿, where φ1φ2∈T (x ). Hence h|f .

Re 4) If g|f , then f =g .h. For the degrees of these polynomials there applies dg f =dgg+dg h, where dg f ≠−∞, dg g≠−∞, dgh≥ 0, so then dg f ≥dg g.

Re 5) If g|f and at the same time f|g then f =g φ1 and also g=f φ2. From the previous part of the proof we know that dg g≤ dg f and also dg f ≤dg g, which means dg f =dg g. Since dg g=dg f +dg φ2, it must be dg φ2=0, so φ2 is a nonzero constant c∈T .

Definition 8 We say that a polynomial h∈T [ x ] is a common divisor of polynomials f , g∈T [ x ] if and only if there apply at the same time both h|g and h|f .

Definition 9 Let f , g , d∈T [ x ]. We call the polynomial d a greatest common divisor of polynomials f , g, if and only if there applies:

1. d|f d|g,

2. For all polynomials h∈T [ x ] there applies: if h|f and also h|g then h|d .

Note: The definition above implies that a greatest common divisor is the common divisor of both polynomials which is divisible by all of the common divisors.

Note: Following there is presented the sequence of equations called Euclidean division of polynomials, which can be used to determine a greatest common divisor of two polynomials. Earlier you have encountered the Euclidean algorithm for finding the greatest common divisor of two integers, which is similar in principle and which will be demonstrated on an example.

Example 15 Use the Euclidean algorithm to find out the greatest common divisor of numbers 450 and 294.

450=294 ∙ 1+156294=156 ∙1+138156=138∙1+18138=18∙7+12

18=12∙ 1+612=6 ∙ 1+0

The greatest common divisor of two numbers is the last nonzero remainder in the Euclidean algorithm, which is D (450 , 294 )=6.

Euclidean Algorithm for Polynomials:

19

Page 20: Polynomials - Pedagogická fakulta MU€¦ · Web viewFrom the algebraic point of view it is especially interesting that the result of adding and multiplying polynomials is again

Let f , g∈T [ x ] , dg f ≥ dg g , g ≠ 0. We start by dividing the polynomial f by the polynomial g and then we repeatedly divide the divisor from the previous step by the remainder, continuing until there is a result with a zero remainder.

f =g . q1+r1 , dg r1<dg g , r1≠ 0 g=r1 . q2+r2 , dg r2<dg r1 , r2 ≠ 0 r1=r2 . q3+r3 , dg r 3<dg r2 ,r3 ≠ 0 ⋮ rk −2=rk−1 . qk+rk , rk ≠ 0 rk −1=rk .qk+1+0

Theorem 7 A greatest common divisor can be found for each pair of polynomials f , g∈T [ x ] , g≠ 0. One of these greatest common divisors is the last non-zero Euclidean algorithm remainder for the polynomials f and g.

Proof: From the previous equation it is apparent from the previous equation that rk|r k−1 , then from the next-to-last equation rk|r k−2 , and so onwards to the second equation from which rk|g , and from the first equation then rk|f , hence rk is a common divisor of the polynomials f , g.

Now it only remains to be proven that every common divisor of polynomials f , g divides rk. If h is a common divisor of both polynomials then it is evident that h|r1 from the first equation, h|r2 from the second one, and so forth to h|r k from the last one.

Note: In accordance with the previous theorem, no greatest common divisor there exists only when f =g=0.

Note: If d1 , d2 are two different common divisors of polynomials f , g, then according to the property 2) in the Theorem 6 there applies d1|d2 d2|d1 and according to 5) there exists a non-zero number c∈T , for which it applies d2=c .d1.

There also applies that when d1 is a greatest common divisor of two polynomials, then d2=c .d1 is their greatest common divisor as well.

Theorem 8 When d∈T [ x ] is a greatest common divisor of polynomials f , g∈T [ x ], then the set {cd|d∈T , c≠ 0 } is the set of all greatest common divisors of the polynomials f , g. We say these common divisors are associated with each other.

Page 21: Polynomials - Pedagogická fakulta MU€¦ · Web viewFrom the algebraic point of view it is especially interesting that the result of adding and multiplying polynomials is again

Among the greatest common divisors of the polynomials f , g there exists exactly one whose coefficient of the highest degree term is equal to 1. This polynomial is called a normalized greatest common divisor and denoted as GCD( f , g).

Example 16 Find the greatest common divisor of following polynomials over Q [ x ]: f ( x )=x4−2 x3−2 x2+7 x−6, g ( x )=2 x3−4 x2−x+2.

Solution: Euclidean algorithm will be applied. Since GCD( f , g) will not change when replacing the polynomials f , g with the polynomials associated with them, we will first multiply f by 2. Then we will divide.

(2 x4−4 x3−4 x2+14 x−12 ): ( 2x3−4 x2−x+2 )=x+−3x2+12 x−122 x3−4 x2−x+2

,

thus q1 ( x )=x , r1 ( x )=−3 x2+12 x−12. Normalizing the polynomial r1 ( x ) into x2−4 x+3 we proceed to the next step, this time dividing g(x ) by r1 ( x ).

(2 x3−4 x2−x+2 ) : ( x2−4 x+4 )=2 x+4+ 7 x−14x2−4 x+3

,

therefore q2 ( x ) 2 x+4 , r2 ( x )=7 x−14. After replacing the polynomial r2(x ) with the normalized polynomial again we can proceed to the division, getting

(x¿¿2−4 x+4) : ( x−2 )=x−2,¿

i.e. q3 ( x )=x−2, r3 ( x )=0. Hence GCD ( f , g )=r2 ( x )=x−2. The discovered greatest common divisor is really over the complex numbers field, thus satisfying the assignment requirement.

Note: In some cases it is also possible to find a greatest common divisor of two polynomials by their decomposition. Let us, for this purpose, revise some basic polynomial decompositions:

a2−b2= (a−b )(a+b), a3−b3=(a−b )(a2+ab+b2), a3+b3=( a+b )(a2−ab+b2), a2± 2 ab+b2=¿,

etc.

Definition 10 A polynomial f ∈T [ x ] is called irreducible4 over a field T if and only if dg f ≥1 and there can be found no polynomials g , h∈T [ x ] such that there applies f =g . h ,dg g≥ 1 , dg h≥1.

4 Any three values of an independent variable can be arbitrarily chosen in any order, but this decision must be respected in the rest of the process and suitable function values must be substituted for f respectively.

21

Page 22: Polynomials - Pedagogická fakulta MU€¦ · Web viewFrom the algebraic point of view it is especially interesting that the result of adding and multiplying polynomials is again

Definition 11 A polynomial f ∈T [ x ] is called reducible over a field T if and only if there exist polynomials g , h∈T [ x ] for which there applies f =g . h , dg g ≥ 1 , dg h≥1.

Note: The role irreducible polynomials play in the integral domain T [ x ] is analogical to the role of prime numbers in the ring Z.

Theorem 9 For each non-constant polynomial f ∈T [ x ] there exists a number a∈T and irreducible polynomials φ1 , …, φk∈T [ x ] over T such that there applies

f =a φ1… φk ,

where φ i, i=1 , …, k are normalized polynomials. This decomposition is unambiguous, except for the ordering of the polynomials φ1 ,…, φk .

The theorem could be also formulated in a different way: Every non-constant polynomial f over a field T can be decomposed into the product of a constant and normalized irreducible polynomials.

Proof: Let f ∈T [ x ]. The polynomial f is irreducible over T , then we normalize it into the form f =a φ1.

Or the polynomial f is reducible. Then there can be found its decomposition into polynomials irreducible over T : f =❑1…❑k. Upon rearranging them into the normalized form, we get f =a φ1… φk.

Now there still remains to prove the unambiguity of the decomposition. The proof will be performed by contradiction. So let us assume that there exist two distinct decompositions of the polynomial f :

f =a φ1… φk=c❑1 …❑k

The polynomials φ i ,❑i are irreducible over T . If the polynomial φ1 divides the polynomial f , then there must also apply φ i¿. Then either φ1=❑1, or the polynomials φ1 ,❑1 are coprime. In the case they are coprime, then among polynomials ❑2 , …,k there must be one that is equal to φ1. Analogically, we can reason for the other polynomials φ2 , …, φk . Thus we reach the contradiction with the assumption that the decompositions are distinct.

Note: When the same irreducible polynomials in the decomposition are multiplied by each other, then

f =a φ1α 1…φr

α r , αi∈N .

This decomposition is called a canonical decomposition.An analogy can be found in the ring of integers. From now on we will consider only

non-negative integers, negative integer could be obtained by multiplication

Page 23: Polynomials - Pedagogická fakulta MU€¦ · Web viewFrom the algebraic point of view it is especially interesting that the result of adding and multiplying polynomials is again

a=(−1)n , n∈ N . Every non-negative integer n greater than 1 can be decomposed into a product of prime factors, n=p1 ∙…∙ pk, i.e.

n=p1α 1… pr

α r .

For instance, number 20 580can be expressed in the form of a canonical decomposition as 22 ∙ 3 ∙5 ∙73.

Theorem 10 Over the field of complex numbers, all polynomials of the 1st degree are irreducible. All polynomials of a degree greater than 1 are reducible.

Theorem 11 Over the field of real numbers, there are irreducible just all polynomials of the 1st degree and those of the 2nd degree that have a negative discriminant.

Theorem 12 For each n∈N there exists a polynomial f ∈Q [ x ] ,dg f =n such that f is irreducible over the field Q.

Example 17 Find decompositions of the following polynomials over Q [ x ] , R [ x ] , C [ x ].a) x2+ x+1,

b) 2 x3+2 x2+x+1,

c) x4+x3+x+1,

d) x6−2.

Solution: a) The polynomial can be decomposed using the procedure well known from a high

school, i.e.

x1,2=−b ±√b2−4ac

2a=−1±√1−4

2=−1±√−3

2.

We can clearly see by now that over both Q [ x ] and R [ x ] the polynomial is irreducible

and over C [ x ] its decomposition is (x−−1−i √32 )(x−−1+i √3

2 ).b) 2 x3+2 x2+x+1=2 x2 (x+1 )+( x+1 )=( x+1 )(2 x2+1), which is the decomposition in both

Q [ x ] and R [ x ]. Over C [ x ] we can continue with rearrangement ( x+1 ) ( 2 x2+1 )=( x+1 ) (2 x2−i2 )= ( x+1 ) (√2 x−i )(√2 x+ i).

c) x4+x3+x+1=x3 ( x+1 )+( x+1 )=( x+1 ) ( x3+1 )=( x+1 ) ( x+1 ) ( x2−x+1 )=( x+1 )2(x2−x+1), which is the decomposition in both Q [ x ] and R [ x ]. In C [ x ] the expression in the last parentheses can be decomposed just like in the part a), thus getting

( x+1 )2(x−1−i √32 )( x−1+i √3

2 ).23

Page 24: Polynomials - Pedagogická fakulta MU€¦ · Web viewFrom the algebraic point of view it is especially interesting that the result of adding and multiplying polynomials is again

d) x6−2=(x¿¿3−√2)(x¿¿3+√2)¿¿. It is apparent that the polynomial is irreducible over Q [ x ]. In R [ x ] it is possible to continue,

(x¿¿3−√2)(x¿¿3+√2)= ( x−3√√2 )( x2+ 3√√2 x+ 3√2 ) (x+ 3√√2 ) (x2− 3√√2x+ 3√2 )¿¿. In C [ x ] further decomposition of the polynomial is possible but we will not perform it due to the form of the expression.

Example 18 Determine a greatest common divisor of polynomials f , g when f =x4−2 x2+1, g= x3+3 x2−x−3

a) using the Euclidean algorithm,b) by decomposition.

Solution:

a) ( x4−2 x2+1 ) : ( x3+3 x2−x−3 )=x−3+ 8x2−8x3+3 x2−x−3

.

Now we normalize the remainder 8 x2−8 into the polynomial x2−1 and continue with the algorithm.

( x3+3 x2−x−3 ) : ( x2−1 )=x+3 remainder. 0.

The last non-zero remainder is 8 x2−8, which is one of greatest common divisors, and GCD ( f , g )= x2−1.

b) In some cases it is possible to find out common divisors and also greatest common divisors by decomposition of the polynomials.

f =x4−2 x2+1=¿,

g= x3+3 x2−x−3=x3−x+3x2−3=x ( x2−1 )+3 ( x2−1 )=( x2−1 ) ( x+3 )= (x−1 ) ( x+1 )(x+3),

GCD ( f , g )= ( x−1 ) ( x+1 )=(x2−1).

Example 19 Solve individually: Apply the Euclidean algorithm to determine GCD ( f , g ) when f =x5−x4+x3−x2+x−1, g= x4−2 x2−3 x−2.

Result: GCD ( f , g )= x2+x+1

Definition 12 Polynomials f , g∈T (x) are called coprime if and only if GCD ( f , g )=1.

Theorem 13 Let f , g∈T [ x ] , g ≠ 0. Then there exist u , v∈T [ x ] such that fu+gv=GCD (f , g).

Furthermore, if dg f ≥1 , dg g ≥1, then polynomials u , v∈T (x) can be found such that dg u<dgg dgv<dg f .

Page 25: Polynomials - Pedagogická fakulta MU€¦ · Web viewFrom the algebraic point of view it is especially interesting that the result of adding and multiplying polynomials is again

Example 20 Use the Euclidean algorithm to determine polynomials u , v such that GCD (f , g )=fu+gv, je-li f =x4+2 x3−x2−4 x−2, g= x4+ x3−x2−2 x−2.

Solution: The first step will be to find GCD( f , g).

(x¿¿4+2 x3−x2−4 x−2): ( x4+x3−x2−2 x−2 )=1+ x3−2 xx4+x3−x2−2 x−2

¿

We will write this result in the form

f =g ∙ q1+r1 , q1=1 , r1=x3−2 x .

Continuing with division,

( x4+x3−x2−2 x−2 ) : ( x3−2 x )=x+1+ x2−2x3−2x

,

We will again rewrite it in the form

g=r1q2+r2 ,q2=x+1 ,r 2=x2−2.

Since x3−2 x=x (x2−2), the last non-zero remainder is the polynomial r2 and hence GCD (f , g )= x2−2.

To determine the sought form of the greatest common divisor r2=fu+gv , we will proceed backwards in the equation sequence

f =g ∙q1+r1

g=r1q2+r2 .

When expressing r2=g−r1 q2 from the second equation and substituting the first equation for r1, we get

r2=g−( f −g q1 ) q2=(−q2 ) f +( 1+q1 q2 ) g ,

whence it is already apparent that u=−q2=−x−1, v=1+q1 q2=x+2.

Overall

x2−2= (−x−1 ) ( x4+2x3−x2−4 x−2 )+( x+2 ) ( x4+x3−x2−2 x−2 ) .Verify the validity of the decomposition by yourselves by expanding the right side of the equation.

Note: When searching for a greatest common divisor of polynomials in the form fu+gv using the Euclidean algorithm, it is not possible to normalize the polynomials or use associated polynomials for the purpose of simplicity.

Theorem 14 Let f , g , h∈T [ x ] . Then there applies

1) If f|gh and at the same time GCD ( f , g )=1, then f|h.

2) If f|h, g|h and GCD ( f , g )=1, then ( fg)|h.

25

Page 26: Polynomials - Pedagogická fakulta MU€¦ · Web viewFrom the algebraic point of view it is especially interesting that the result of adding and multiplying polynomials is again

Proof: Re 1: We know from the previous theorem that there exist polynomials u , v∈T [ x ] such that fu+gv=GCD (f , g), then obviously

fu+gv=1.

When multiplying this equation with the polynomial h, we getfhu+ghv=h .

It is apparent that f|f and according to the initial assumption f|gh , therefore from the previous equation there must also apply that f|h.

Re 2: If f|h, then there exists h1∈T [ x ] such that h=f h1. Since by the initial assumption also g|h, then after the substitution for h from the previous step, there also applies g|f h1. Furthermore, we assume that GCD ( f , g )=1, so we can employ the first part of the theorem, resulting into

g|h1∃h2∈T [ x ] : h1=g h1 h= fgh2( fg)|h

Page 27: Polynomials - Pedagogická fakulta MU€¦ · Web viewFrom the algebraic point of view it is especially interesting that the result of adding and multiplying polynomials is again

Exercises for Check

1. Define polynomial using the algebraic definition. Demonstrate how from this form there can be obtained the form you are familiar with from high school.

2. State the theorem on polynomial division with a remainder.3. Define the Taylor polynomial.4. Describe the Euclidean algorithm for division of polynomials.5. Define common divisor of two polynomials and greatest common divisor of two

polynomials.6. While calculating the value of the polynomial, a student pressed the following buttons

on a calculator:7 ×6=× 6=−2=×6=−3=×6=+1=× 6=+2=×6=¿

Determine the polynomial the student was evaluating and the value of x. 7. Verify that there applies t 4+t 2−3 t+7=(t 3+3 t2+10 t+27 ) (t−3 )+88. Assemble a

Horner scheme corresponding to this equation.8. Divide the polynomial −5 x4+4 x2−3 x+4 by the polynomial x3+2 x2−4.9. Find out a polynomial that passes through the points [−1 ;−7 ] , [ 0 ;−3 ] , [ 1;−1 ] , [2 ;11].10. Find a greatest common divisor of the polynomials f =x4−x3−2x2+x+1,

g= x3−x2−x+1 by both Euclidean algorithm and decomposition.

Results:

6. 7 x6−2 x4−3 x3+x2+2 x, c=67.

1 0 1 -3 7

3 1 3 10 27 88

8. −5 x+10+−16 x2−23 x+44x3+2x2−4

9. 2 x3−x2+x−310. x2−1

27

Page 28: Polynomials - Pedagogická fakulta MU€¦ · Web viewFrom the algebraic point of view it is especially interesting that the result of adding and multiplying polynomials is again

3 Algebraic Equations

Chapter Objectives

Having studied this chapter you will be able to: define algebraic equation find roots of a polynomial review the procedures for solving of linear and quadratic equation generically solve equations of 3rd degree solve binomial and reciprocal equations

Time to Study

Circa 3 hours

Study Guide

Definition 13 By an algebraic equation we mean an equation in the form

an xn+an−1 xn−1…a1 x+a0 ¿0 ,

where a i R for i=0 ,1 ,... , n.

Note: Solving an algebraic equation stands for searching for roots of the polynomial on the left side of the equation. In the following text there will be demonstrated procedures for solving of selected cases of algebraic equations as well as algebraic inequalities.

This text is focused solely on the equations whose coefficients are real numbers. There also exist equations with complex coefficients, for which the polynomial on their left side is over the field of complex numbers. However, since you will not usually encounter these polynomials in your common practice, this text is limited to polynomials over the field of real numbers.

Yet it is worth reminding that a polynomial with real coefficients can have both real and complex roots.

Page 29: Polynomials - Pedagogická fakulta MU€¦ · Web viewFrom the algebraic point of view it is especially interesting that the result of adding and multiplying polynomials is again

3.1 Roots of Polynomial

Definition 14 A number c∈T is called a root of a polynomial f ∈T [ x ], if and only if f ( c )=0.

Theorem 15 (Bézout’s Theorem): Number c∈T is a root of a polynomial f ∈T [ x ] if and only if

(x−c)|f .

Proof: For any c∈T and an arbitrary f ∈T [ x ] there applies f =( x−c ) g+r .

1) c is a root, then f ( c )=0 r=0 f = (x−c ) g(x−c)|f2) ( x−c )|f f =( x−c )h , h∈T [ x ] f (c )=0c is a root.

Definition 15 A number c∈T is called a k-fold multiple root of a polynomial f ∈T [ x ] , k∈N if and only if there applies:

1) ¿

2) ¿ does not divide f .

Example 21 What is the root multiplicity of number 2 for the polynomial f ( x )=x5−5x4+7 x3−2x2+4 x−8?

Solution: The Horner scheme will be used:

1 -5 7 -2 4 -8

2 1 -3 1 0 4 02 1 -1 -1 -2 02 1 1 1 02 1 3 7

Thus number 2 is a 3-fold multiple root of the polynomial f and there applies

f ( x )=x5−5 x4+7 x3−2x2+4 x−8=( x−2 ) ( x4−3 x3+x2+4 )=¿

Example 22 Find the value of a in the polynomial f =x5−a x2−ax+1 such that the number c=−1 is its at least a 2-fold multiple root.

Solution:

29

Page 30: Polynomials - Pedagogická fakulta MU€¦ · Web viewFrom the algebraic point of view it is especially interesting that the result of adding and multiplying polynomials is again

1 0 0 -a -a 1

-1 1 -1 1 -(1+a) 1 0

-1 1 -2 3 -(4+a) 5+a

There must apply that 5+a=0, hence a=−5. When actually dividing the polynomial x5+5 x2+5 x+1 by the polynomial ¿, we then get

x5+5 x2+5 x+1=¿

Note: When searching for roots of algebraic equations, it might be useful to apply the following theorem, which is valid for algebraic equations with real coefficients.

Theorem 16 Let

an xn+an−1 xn−1…a1 x+a0 ¿0

be an algebraic equation. Let x1 , ... , xn be its roots (both real and complex).

Let A=max {a i; i=0 ,... , n1 }. Then there applies:

1. For each index i ,i=1 , 2 ,... , n there applies the inequality |x i|<1+ A|an|.

2. The number of positive real roots is equal to the number of sign changes in the sequence of non-zero coefficients an , an−1 , …, a0 or it is lesser by an even number.

Implication: When all coefficients of a polynomial are positive real numbers, this polynomial has no positive roots.

Example 23 Determine intervals in which there lie real roots of the equation 2 x5+3 x4−7 x3+6 x2−11 x+5=0.

Solution: an=2 , A=11, thus x i∈1+ A|an|

=1+ 112

=132 . Hence all roots of the equation lie in

the interval x i∈(−132

, 132 ).

There are four sign changes in the sequence of coefficients 2, 3, -7, 6, -11, 5, thus in

the interval (0 , 132 ) there lie either 4 or 2 roots.

Definition 16 We call a field T to be algebraically closed if and only if every polynomial f ∈T [ x ] with the degree of 1 or greater has at least one root over the field T .

Page 31: Polynomials - Pedagogická fakulta MU€¦ · Web viewFrom the algebraic point of view it is especially interesting that the result of adding and multiplying polynomials is again

Example 24 The field of rational numbers is not algebraically closed. For example the polynomial f =x2−2 has rational coefficients, its degree is 2, but its roots k=±√2 do not belong into the set of rational numbers. Similarly, the field of real numbers is not algebraically closed. For instance, the polynomial f =x2+1 has real coefficients, its degree is 2, but it does not have real roots because k=± i.

Theorem 17 (Fundamental Theorem of Algebra): Every polynomial with complex coefficients and with the degree of 1 or greater has at least one complex root.

The fundamental theorem of algebra can be also formulated as follows: The field of complex numbers is algebraically closed.

Note: All known proofs of the Fundamental theorem of algebra are based on methods of mathematical analysis. Performing the proof would require application of some theorems from mathematical analysis, whose implication is the fundamental theorem of algebra. Therefore no proof will be included.

Theorem 18 Let f ∈T [ x ] , dg f =n ≥ 1. Then the polynomial f has n roots over C, provided that a k−¿ fold multiple root is counted as k roots. Therefore for each polynomial f ∈T [ x ] , dg f ≥ 1 there exist numbers a , α 1, …,α n∈C such that f =a ( x−α1 ) ( x−α2 ) … ( x−α n ) .

Proof: According to the Fundamental theorem of algebra, there exists a root α 1 such that f =( x−α1 ) f 1, dg f 1=n−1. If n−1≥ 1, then there exists a root α 2 of a polynomial f 1 such that f 1=( x−α2 ) f 2, so f =( x−α1 )(x−α2) f 2, and so on. Eventually f =( x−α1 ) …(x−αn) f n, dg f n=0, thus the polynomial f n is a constant one, f n=a.

Theorem 19 Let f ∈ R [ x ] , c∈C , f (c )=0. Then the number c∈C is also a root of the polynomial f and its multiplicity is the same as of the root c.5

Proof: 1) Let c be a root of a polynomial f =a0 xn+…+an. Then f ( c )=a0 cn+…+an=0. There only remains to be proven that f (c )=f (c ) :

0=f (c)=a0 cn+…+an=a0 cn+…+an=a0 . cn+…+an=¿

¿a0¿

Hence c is a root of the polynomial f as well.2) Let c be a k−¿ fold multiple root of a polynomial f ,k∈N ,c R , c≠ c. We will

prove that c is also a k−¿ fold multiple root.

f =( x−c ) f 1=( x−c ) ( x−c ) f 2=[ x2−( c+c ) x+c c ] f 2

5 Irreducible stands for one that cannot be decomposed into a product of non-constant polynomials.

31

Page 32: Polynomials - Pedagogická fakulta MU€¦ · Web viewFrom the algebraic point of view it is especially interesting that the result of adding and multiplying polynomials is again

As c+c=2a and c c=a2+b2, the polynomial x2−(c+c ) x+cc and thus also the polynomial f 2 has real coefficients.

f =( x−c ) ( x−c ) f 2=( x−c ) (x−c ) ( x−c ) ( x−c ) f 4=…=¿

where the polynomial h cannot have roots c , c. Therefore c is a k−¿ fold multiple root.

3.2 Polynomial Derivative

Definition 17 Let f ∈T ( x ) , f =a0 xn+a1 xn−1+…+an−1 x+an. Then the polynomial f '=n a0 xn−1+ (n−1 ) a1 xn−2+…+an−1 is called a derivative of the polynomial f .6

Theorem 20 For the derivative of polynomials f , g∈T [ x ] there applies:1) ( f ± g )'=f ' ± g'

2) ( f . g )'=f ' . g+g' . f3) k∈N , ( f k )'=k . f k−1 . f '

4) c '=0 for c∈T

Note: The proof of this theorem has been performed in detail in mathematical analysis.

Theorem 21 Let k∈N ,k ≥ 2 , c∈T , f ∈T [x ]. Then there applies: When the number c is a k-fold multiple root of the polynomial f , then the number c is a (k−1)- fold multiple root of the polynomial f ' . When c is a simple root of the polynomial f then c is not a root of the polynomial f ' .

Proof: Let c be a k-fold multiple root of a polynomial f . Then there applies¿.

For the derivative of the polynomial f there applies

f '=k ¿¿.

We will prove by contradiction that c is not a k-fold multiple root of the polynomial f ' . If ¿ divided f ' , then there would apply f '=¿. From the previous assumption ¿ for the derivative there would have to apply

¿.After cancellation we get

6 Let us remind what a number c, called a complex conjugate of the number c, stands for. When c=a+bi , a ,b∈ R, then c=a−bi. Furthermore, for numbers c1 , c2∈C there applies: c1 ±c2=c1 ± c2, c1 .c2=c1 . c2, for k∈N : z1

k=¿, for a∈R : a=a

Page 33: Polynomials - Pedagogická fakulta MU€¦ · Web viewFrom the algebraic point of view it is especially interesting that the result of adding and multiplying polynomials is again

( x−c ) h1=kh+( x−c ) h '.

Choosing x=c we get after substitution0=k .h(c ).

Since k is independent on c then h (c )=0, which means that h=( x−c ) g , g∈T [ x] andf =¿.

That would imply that the number c is a (k+1)-fold multiple root of the polynomial f , which is a contradiction with the initial assumption that c is only a k-fold multiple root of the polynomial f .

Note: The theorem is not valid in the opposite way: If a number c is a root of the polynomial f ' , then it does not necessarily have to be a root of the polynomial f . For example. f =¿, f (3)≠ 0, f '=5¿ and number 3 is a 4-fold multiple root of the polynomial f ' .

Example 25 Given the polynomial f =a x4+b x3+1, find coefficients a ,b such that 1 is at least a double root.

Solution: If 1 is a double root of the polynomial f then it is a simple root of the polynomial f ´ . Therefore we will find the derivative of the polynomial f and determine the values of the polynomials f , f ´ at the point 1:

f ´ ( x )=4 a x3+3 b x2

f (1 )=a+b+1

f ´ (1 )=4 a+3b

Now all that remains is to solve the following system of equations:a+b+1=04 a+3 b=0

Having solved this system of equations, we learn that a=3 , b=−4, so the form of the polynomial f is

f =3 x4−4 x3+1.

Example 26 Solve Example 22 using the derivative.

Solution:

f ´ ( x )=5 x4−2 ax−a, f (−1 )=−1−a+a+1=0, f ´ (−1 )=5+2 a−a=5+a, f ´ (−1 )=0a=−5. The result is the polynomial f =x5+5 x2+5 x+1.

Theorem 22 Let k∈N , k ≥ 2 , c∈T , f ∈T [ x ]. Then the following propositions are equivalent to each other:

1) c is a k−¿fold multiple root of the polynomial f.

33

Page 34: Polynomials - Pedagogická fakulta MU€¦ · Web viewFrom the algebraic point of view it is especially interesting that the result of adding and multiplying polynomials is again

2) ¿and at the same time ¿ does not divide f .

3) There exists a polynomial g∈T [ x ] such that f =¿ and at the same time g(c)≠ 0.

4) f ( c )=0 , f ´ (c )=0 , f ´ ´ ( c )=0 , …, f ( k−1) ( c )=0 , f (k)≠0.

5) c is a (k−1 )−¿fold multiple root of the polynomial d=NSD ( f , f ´ ).

Note: When searching for multiple roots of a polynomial f , we can adopt the following procedure:

1. Find GCD (f , f ´ )=d .

2. If d=1, then f does not have any multiple roots.3. If d ≠1, then each root of the polynomial d is also a root of the polynomial f , and

moreover it is its multiple root: its multiplicity is greater by 1 than its multiplicity for the polynomial d .

Example 27 Demonstrate that the polynomial f ( x )=x3+x2−5 x−2∈R [ x ] has only simple roots.

Solution: We will proceed according to the procedure in the previous note. Seeking the derivative of the polynomial f , we find it to be f ´ ( x )=3 x2+2x−5. You can verify for yourselves by application of the Euclidean algorithm that GCD ( f , f ´ )=1. Thus the polynomial f ( x ) has truly only simple roots.

Theorem 23 Let f ∈T [ x ] , d=GCD ( f , f ´ ) , f =d . g. Then there applies: The polynomial g has the same roots as the polynomial f and all of the roots of the polynomial g are simple.

Example 28 Let the polynomials for example be f =¿, d=¿. Then the polynomial g= ( x−1 ) ( x+2 )(x+7) and f =d . g.

3.3 Polynomials with Integer Coefficients

Theorem 24 Let p∈Z , p ≠0 , q∈N ,GCD ( p ,q )=1 ,c= pq

, f =an xn+…+a0∈Z [ x ].

When c is a root of the polynomial f , then for any m∈Z there applies:1) p|a0,

2) q|an ,

3) ( p−mq)|f (m).

Page 35: Polynomials - Pedagogická fakulta MU€¦ · Web viewFrom the algebraic point of view it is especially interesting that the result of adding and multiplying polynomials is again

Proof: c=pq is a root of the polynomial, i.e. f ( c )=0. Hence for the polynomial f we can

write

an( pq )

n

+an−1( pq )

n−1

+…+a0=0.

When multiplying this equation by qn, we get

an pn+an−1 pn−1 q+… a1 p qn−1+a0 qn=0.

1) Having transposed the last term a0qn to the right side of the equation and factored out p from all previous terms, we get:

p [an pn−1+…+a1qn−1 ]=−a0 qn ,

thus p|a0qn, and as both p and q are coprime, there must apply p|a0.

2) Similarly as in the previous step we can transpose the term an pn to the right side of the equation and factor out q from all following terms, so getting:

q [ an−1 pn−1+…+a0 qn−1 ]=−an pn ,

reasoning similarly as in step 1), we can conclude that q|an .

3) The polynomial f can be expressed in the form of the Taylor polynomial centred at m:

f =c0+c1 ( x−m )+…+cn−1 (x−m )n−1+cn ( x−m )n .

It is obvious that f (m)=c0 . When we now perform substitution of x with the root

c= pq , we get

f ( c )=c0+c1( p−mqq )+…+cn−1( p−mq

q )n−1

+cn( p−mqq )

n

=0 ,

after multiplication of the equation by the expression qn we get

c0 qn+c1 ( p−mq )qn−1+…+cn−1 ( p−mq )n−1 q+cn ( p−mq )n=0.

Reasoning the same way as in the steps 1) and 2), we can come to the conclusion that ( p−mq)|c0 qn, and since GCD ( p−mq ,q )=1, there applies ( p−mq)|c0 , that is ( p−mq)|f (m).

Note: The last theorem can be used to find all rational roots of polynomials with integer coefficients. In fact, the application of the part 3) of the last theorem consists in choosing certain values for m, which are numbers 1and −1, thus

( p−q )|f (1 ) ( p+q )|f (−1 ) .

Example 29 Find all rational roots of the polynomial f =2 x5+x4−12 x3−20 x2−19 x−6.

35

Page 36: Polynomials - Pedagogická fakulta MU€¦ · Web viewFrom the algebraic point of view it is especially interesting that the result of adding and multiplying polynomials is again

Solution: The roots being searched for will have the form c=pq where p|(−6), q|2.

Divisors of number −6 :1,2,3,6 ,−1,−2 ,−3 ,−6.Positive divisors of number 2 :1,2.

Next we will create a table where in the first row there will be all possible values of pq , corresponding values of p+q will be in the second row, and p−q in the third one.

Since f (−1 )=4, we will highlight all divisors of number 4 in the second row, and because of f (1 )=−54, we will similarly highlight all divisors of number −54 in the third row.

Candidates for roots will then be those numbers pq which will have both second and third

row values highlighted.

pq

11

12

21

31

32

61

−11

−12

−21

−31

−32

−61

p+q 2 3 3 4 5 7 0 1 -1 -2 -1 -5p−q 0 -1 1 2 1 5 -2 -3 -3 -4 -5 -7

Therefore the candidates for roots are the numbers: 3 ,−12

,−2. We will verify these roots

using Horner’s method:2 1 -12 -20 -19 -6

−12

2 0 -12 -14 -12 0

3 2 6 6 4 0

−2 2 2 2 0

We can clearly see in the Horner scheme that

f =(x+ 12 ) . (2 x4−12 x2−14 x−12 )=(2 x+1 ) ( x4−6 x2−7 x−6 )=(2 x+1 ) ( x−3 ) ( x3+3 x2+3 x+2 )= (2 x+1 ) ( x−3 ) ( x+2 )(x2+x+1)

The polynomial x2+ x+1 has no real roots and all sought rational roots are therefore

numbers 3 ,−12

,−2.

Example 30 Solve individually: Find all rational roots of the polynomial f =6 x4+19 x3−7 x2−26 x+12.

Result: Candidates for roots: 2 , 12

,−3 ,−13 , f =( x+3 ) (2 x−1 )(3x2+2 x−4), rational roots

are −3 , 12 .

Page 37: Polynomials - Pedagogická fakulta MU€¦ · Web viewFrom the algebraic point of view it is especially interesting that the result of adding and multiplying polynomials is again

Note: The previous procedure can be employed to solving certain algebraic inequalities as well, as shown in the following example.

Example 31 Solve the inequality x3+4 x2+x−6 ≤ 0.

Solution: A number p is a divisor of number −6, a number q is a divisor of 1. When substituting number 1 into the expression on the left side of the inequality, we find out that the value is 0. We will term the left side of the inequality as f (x) and the discovered fact as f (1 )=0, which means that 1 is a root. For this reason we will not include into a table a row for p−q, because divisors of zero cannot be found. Further on f (−1 )=4.

pq

1 2 3 6 -1 -2 -3 -6

p+q 2 3 4 7 0 -1 -2 -5

After checking the candidates −3 ,−2, 1 ,3 using Horner’s method, we find that the numbers −3 ,−2, 1 are roots of the polynomial, and the assigned inequality can be rewritten into the form

( x−1 ) ( x+2 )(x+3)≤ 0.

Now we can proceed on using the number line and zero points (perform by yourselves). The result are all numbers x∈ ¿.

3.4 Vieta’s Formulas

Certain problems can be solved by application of relationships between roots and coefficients of a polynomial that are presented in the following theorem.

Theorem 25 Let r1 , r2 ,…, rn∈T be all roots of a polynomial f ∈T [ x ] , dg f =n , f =an xn+…a1 x+a0. Then there apply so-called Vieta’s formulas:

−an−1

an=r1+r2+…+rn

an−2

an=r1 r2+r1r 3+…+rn−1 rn

¿¿

37

Page 38: Polynomials - Pedagogická fakulta MU€¦ · Web viewFrom the algebraic point of view it is especially interesting that the result of adding and multiplying polynomials is again

Proof: When r1 , r2 ,…, rn are roots of the polynomial f , there applies an xn+…a1 x+a0=an ( x−r1 ) ( x−r2 ) …(x−rn). As we expand the right side and compare the coefficients for the same powers of x, we get Vieta’s formulas.

Example 32 Apply Vieta’s formulas to find a polynomial whose roots are −1 ,−1 ,−1,3,4 .

Solution: There are five roots, thus we know that n=5. The coefficient a5 can be arbitrarily chosen, so we choose a5=1. Then

−a4

1=−1−1−1+3+4=4

a3

1=1+1−3−4+1−3−4−3−4+12=−6

¿¿¿

After rearrangement we get a0=12 , a1=29 , a2=16 , a3=−6 ,a4=−4 , a5=1. The polynomial f has the form

f =x5−4 x4−6 x3+16 x2+29 x+12.We can verify the validity by expanding:

f =an¿

Example 33 Solve individually: Apply Vieta’s formulas to find a polynomial whose roots are i ,i ,−1−i.

Result: f =x3+(1−i ) x2+(1−2 i ) x−1−i

Note: Vieta’s formulas can be utilised for simple in-memory solving of some quadratic

equations with integer roots. For an equation ax2+bx+c=0 there applies r1+r2=−ba ,

r1 . r2=ca . For example, in case of the polynomial x2+5 x+6, for its roots there applies

r1+r2=−5 and r1 . r2=6. Upon solving this system of equations, we get r1=−2, r2=−3.

Example 34 Find out the coefficients and the form of a quadratic polynomial a x2+bx+c

knowing that the product of its roots is equal to −52 and its sum to

−32 .

Solution: According to Vieta’s formulas there applies:

Page 39: Polynomials - Pedagogická fakulta MU€¦ · Web viewFrom the algebraic point of view it is especially interesting that the result of adding and multiplying polynomials is again

1) x1+ x2=−ba

2) x1 x2=ca

Hence we know that −ba

=−32 and

ca=−5

2 . The form of the fractions implies that a=2,

then b=3 and c=−5. Thus the form of the sought polynomial is 2 x2+3 x−5.

Example 35 There is given the quadratic equation x2+11x+19=0. Let us mark its two distinct real roots with the symbols x1 , x2. Compute the value of the expression x1

2+ x22.

Solution: We will apply the knowledge of Vieta’s formulas

1) x1+ x2=−ba ,

2) x1 x2=ca ,

thus x1+ x2=−11, x1 x2=19. When rearranging the expression ( x1+x2 )2=x12+2 x1 x2+ x2

2, we

get x12+ x2

2=( x1+x2 )2−2 x1 x2, and after substitution x12+ x2

2=(−11)2−2.19=83.

It was possible to solve the example also without Vieta’s formulas, computing the roots of the quadratic equation using the standard formula, but considering that the roots are not rational, the process of computation could prove to be rather difficult indeed, as you can check by yourselves.

Example 36 Find all roots of the polynomial x4−4 x3+5 x2−2 x−6 given the knowledge that for the roots of the polynomial there applies the relation x1+ x2=x3+x4.

Solution: According to Vieta’s formulas there applies:

1) x1+ x2+x3+x4=4

2) x1 x2+x1 x3+ x1 x4+x2 x3+ x2 x4+x3 x4=5

3) x1 x2 x3+x1 x2 x4+x1 x3 x4+x2 x3 x 4=2

4) x1 x2 x3 x4=−6

When substituting the relation x1+ x2=x3+x4 into the first equation, we get

2 ( x1+x2 )=4 x1+x2=2 , x3+x4=2

From the second equation we get by rearrangementx1 x2+x1(x¿¿3+x4)+x2(x¿¿3+ x4)+x3 x4=x1 x2+( x3+x4 ) . ( x1+x2 )+x3 x4=x1 x2+2.2+x3 x4=5 x1 x2+x3 x4=1¿¿

Combining this rearranged equation with the fourth equation:

2) x1 x2+x3 x4=1

4) ( x1 x2 ) . ( x3 x4 )=−6

39

Page 40: Polynomials - Pedagogická fakulta MU€¦ · Web viewFrom the algebraic point of view it is especially interesting that the result of adding and multiplying polynomials is again

Now we perform the substitution x1 x2=u x3 x4=1−u u (1−u )=−6

u2−u−6=0u1=3 ,u2=−2

Returning to the original variables, we get x1 x2=3 , x3 x4=−2.

Now we will solve the system of two equations x1 x2=3 and x1+ x2=2, perform the substitution x1=v x2=2−v, and having solved the equation

v2−2 v+3=0

we come to two roots x1=1+i √2 , x2=1−i √2.

When we use an analogical process for the equations x3 x4=−2 and x3+ x4=2, we get another two roots x3=1+√3 , x4=1−√3.

As we are dealing with a polynomial of the fourth degree, which has four roots, we have found all of its roots.

Example 37 Solve individually: Find all roots of the polynomial x4+2 x3+2 x2+10 x+25=0, when among the roots of the polynomial there applies the relation x1 x2=x3 x4.

Result: x1=−2−i , x2=−2+i , x3=1+2 i , x4=1−2 i

Following are briefly described basic methods of solving some selected types of algebraic equations.

3.5 Linear Equation

The term stands for an equation in the form x+b=0 , a≠0 . This equation has always a solution in the form

x=−ba

.

3.6 Quadratic Equation

It is an equation in the form a x2+bx+c=0 , a ≠ 0. You have already encountered this type of equation at elementary school where you were solving quadratic equations in a special form. You learned to generically solve quadratic equations at high school. The following text reminds how quadratic equations are being solved.

Page 41: Polynomials - Pedagogická fakulta MU€¦ · Web viewFrom the algebraic point of view it is especially interesting that the result of adding and multiplying polynomials is again

For the calculation of roots a quadratic equation is rearranged using the method of the complement to a full square.

a x2+bx+c=a [ x2+px+q ]=a[(x+ p2 )

2

− p2

4+q ]

Now if we mark the expression p2

4−q with the letter d , from the formula for difference of

squares we get

a [(x+ p2 )

2

−d ]=a(x+ p2−√d)(x+ p

2+√d )=0

Discussing the last term: if the expression a (x+ p2−√d )(x+ p

2+√d) is to be zero, then

either one or another of the expressions in parentheses must be zero, thus

(x+ p2−√d)=0(x+ p

2+√d)=0.

As p=ba , d= p2

4−q= b2

4 a2 −ca=b2−4ac

4 a2 , for roots of quadratic equation we get the well-

known formula

x1,2=−b ±√b2−4ac

2a.

The expression b2−4 ac=D is called a discriminant and according to it being positive or negative the decision can be promptly made whether the equation has real or complex roots.

Let us just remind that in some simple cases with integer coefficients Vieta’s formulas are more suitable to be used to determine roots faster. For instance, for the equation x2+7 x+10=0 there applies x1 . x2=10 , x1+x2=−7, so the roots are x1=−2 , x2=−5.

3.7 Cubic Equation

The cubic equation is an equation in the form a3 x3+a2 x2+a1 x+a0=0 , a3 ≠ 0. The first step in its solution procedure consists of dividing the equation by the number a3 and after re-labelling of the coefficients we get the following equation

x3+a x2+bx+c=0

We proceed by using substitution x=t . After rearrangement, we get the equation in the so-called reduced form

t 3+ p t +q=0

41

Page 42: Polynomials - Pedagogická fakulta MU€¦ · Web viewFrom the algebraic point of view it is especially interesting that the result of adding and multiplying polynomials is again

Let us mark ε=cos 2 π3

+isin 2 π3

=12+ i √3

2. Furthermore, let K=√ q2

4+ p3

27 denote one

(fixedly chosen) of the two values of the second square root. Next, let u denote any (fixed)

of the three cube roots 3√−q2

+K , and lastly, let v denote one of cube roots 3√−q2

−K for

which there applies the relationship 3 uv=p. Then roots of the equation t 3+ p t +q=0 are

t 1=u+v , t 2=. u+2 . v , t 3=❑2 .u+ . v .

The previous procedure is called Cardan’s method. A cubic equation can have both real and complex roots. Their nature can be

determined from the value of a discriminant which has the form D=4 p3 27 q2. When being limited to a cubic equation t 3+ p t +q=0 with real non-zero coefficients (otherwise, when p=0 or q=0, the solution is trivial).

a) D=0: t 1=−2 3√ q2

, t2=t3=3√ q

2.

b) D 0: one real root and two imaginary complex conjugate roots, specified by the Cardan formula.

c) D 0: three real roots, however in this case it is not possible to find them using Cardan’s method (so-called “cassus irreducibilis”). The goniometrical solution procedure needs to be applied:

Firstly, we compute the value of the angle φ (one fixedly chosen) from the equation

cos φ=

−q2

√(|p|3 )

3 . The sought roots are then specified by the following relationships:

t 1=2√(|p|3 )cos φ

3,t 2=−2√(|p|

3 )cos ( φ3−π

3 ) ,t 3=−2√(|p|3 )cos ( φ

3+ π

3 ).It is obvious that both Cardan’s method and the lastly-mentioned relationships are

very laborious and cumbersome for practical calculations. Moreover, it is rather difficult to rearrange the acquired results into a usable form. For our purposes it is therefore more suitable to find the solution of a cubic equation in the way described in Chapter 3.3 which allows determining roots of an equation with integer coefficients.

3.8 Fourth Degree Equation

It is an equation in the form a4 x4+a3 x3+a2 x2+a1 x+a0=0 , a4 ≠ 0. This equation is of the last degree that is generally solvable. Nevertheless, the general solution procedure for a 4th

degree equation is toilsome and tedious to that extent that it is hardly ever used in practice, and thus it will not be presented in this text. For equations in the school practice some

Page 43: Polynomials - Pedagogická fakulta MU€¦ · Web viewFrom the algebraic point of view it is especially interesting that the result of adding and multiplying polynomials is again

simpler ways of solving are sufficient (transformation of an biquadratic equation in the form a x4+b x2+c=0 by the substitution to a quadratic equation, procedures for solving binomial and reciprocal equations).

3.9 Higher Degree Equations

For equations of the 5th and higher degree there does not exist any generic solving algorithm anymore 7. According to the theory created by French mathematician Galois there exists for every n 5 an algebraic equation of the degree n that is not solvable by algebraic methods. Even though many equations of higher degrees can be solved, they must be in some special form. Either it is possible to “guess” the roots one at a time, or the equation is in the form that allows it to be solved as a binomial or reciprocal equation. Both these methods are being covered at high school and will be reminded in the following chapters.

Another method how to obtain roots of an algebraic equation, is to use numerical methods. Advances in information and computation technology contribute to the widespread use of these methods. The general procedure consists of the following three steps: assessing bounds of polynomial roots (according to Theorem 16), their separation, and approximation. Searching for roots of polynomials with real coefficients by applying these numerical methods relies on the observation that a polynomial is a continuous function in the domain of real numbers. Among contemporarily used methods there belong the Bisection method, the Fixed-point iteration, Newton’s method and Halley’s method. These problems will not be dealt with in the following text.

3.10 Selected Types of Algebraic Equations

Binomial Equation

A binomial equation stands for an equation of the type xn=a ,n∈N , a∈C. According to the Fundamental theorem of algebra there exist n solutions of this equation. These solutions are in the form

x=n√|a|[cos α+2 kπn

+i sin α +2 kπn ]

and they form vertices of a regular planar n−¿gon. This n−¿gon lies in the Gauss plane on a circle that has its centre in the origin of coordinate axes and its radius r=n√|a|. When searching for these roots, we proceed as follows:

7 You have already encountered the derivative during your study of mathematical analysis. The derivative was defined generically for a function and could be applied to analyze various functions’ properties, such as local extremes of function, convexity and concavity, examination of its behavior and so on.

43

Page 44: Polynomials - Pedagogická fakulta MU€¦ · Web viewFrom the algebraic point of view it is especially interesting that the result of adding and multiplying polynomials is again

1. Write down the sought root in the form xk=r (cosφ+i sin φ).

2. Find the radius r=n√|a|∈R.

3. Find the angle φ for which there applies nφ=α+2kπ , k∈Z , thus φ=αn

+ 2kπn ,

where k=0 , 1 ,…,n−1. The angle α can be then found from the position of the number a in the Gauss plane. It is the angle that the position vector of the number a contains with the real axis.

Example 38 Solve the equation x3−2=0 in C. Depict its roots in the Gauss plane of coordinates.

Solution: First we rewrite the equation in the form x3=2.

1) We will write the sought roots in the form xk=r (cos φ+isin φ ).

2) r=3√2.

3) α=0 or α=2 π, since number 2 lies in the Gauss plane on the positive part of the

real axis. Then φ=03+ 2kπ

3=2kπ

3,k=0 , 1 ,2.

The solution are three roots in the following form:

x0=3√2 (cos0+i sin 0 )= 3√2

x1=3√2(cos 2π

3+isin 2π

3 )x2=

3√2(cos 4 π3

+i sin 4 π3 )

We can see that the first root is a real one while the other two are complex. They form vertices of a regular triangle in the Gauss plane.

Re

Img

23

Example 39 Solve the equation x5=2−2i.

Page 45: Polynomials - Pedagogická fakulta MU€¦ · Web viewFrom the algebraic point of view it is especially interesting that the result of adding and multiplying polynomials is again

Solution: When finding a radius for the number z=a± bi, there applies

|z|=√a2+b2 , r=n√√a2+b2. Therefore, in our example there applies

|z|=√4+4=√8=2√2 ,r=5√2√2.

The number 2−2i lies in the fourth quadrant, in which case the angle α can be determined as α=2 π−α 0, where α 0 can be computed from the following right triangle:

Re

Img

2

2a0

α 0=π4

, α=2π−π4=7 π

4 . There are five roots for which there applies

xk=5√2√2(cos

7 π4

+2kπ

5+i sin

7 π4

+2 kπ

5 ), k=0,1,2,3,4.

Example 40 Solve the following binomial equations individually:1) x3−27=02) x3=4√3+4 i3) x4+16=0

Results:

1) x0=3 , x1=3(cos 2 π3

+i sin 2 π3 ) , x2=3 (cos 4 π

3+isin 4 π

3 )2) x0=2(cos π

18+i sin π

18 ) , x1=2(cos 13π18

+i sin 13 π18 ) , x2=2(cos 25 π

18+isin 25 π

18 )3)

x0=2(cos π4

+isin π4 ) , x1=2(cos 3 π

4+ isin 3 π

4 ) , x2=2(cos 5 π4

+i sin 5 π4 ) , x3=2(cos 7 π

4+i sin 7 π

4 )

Reciprocal Equation

A reciprocal equation is an equation of the form

an xn+an−1 xn−1…a1 x+a0=0 ,

where an−i=ai for i=1,2 , …, n, or an−i=−ai for i=1,2 , …, n.

45

Page 46: Polynomials - Pedagogická fakulta MU€¦ · Web viewFrom the algebraic point of view it is especially interesting that the result of adding and multiplying polynomials is again

It is called reciprocal on the account of the observation that when c is one of its roots then 1c (the reciprocal value) is also its root.

The reciprocal equation of the even degree for an even n can be distinguished from the one of the odd degree for an odd n, just as the equation of the 1st class when an−i=ai, from the equation of 2nd class when an−i=−ai.

The procedure for solving a reciprocal equation is as follows: Each reciprocal equation of the 2nd class has the root c=1. Upon dividing it by the

binomial (x−1), we get a reciprocal equation of the 1st class. Each reciprocal equation of the 1st class of an odd degree has the root c=−1. When

divided by the binomial (x+1), the result is a 1st class reciprocal equation of an even degree.

A 1st class reciprocal equation of an even degree can be transformed into an

algebraic equation of the half degree by dividing it by the expression xn2, and

performing the substitution

y=x+ 1x

.

Example 41 Find roots of the equation 2 x7+5 x6−3 x4+3 x3−5 x−2=0.

Solution: It is a 2nd class reciprocal equation of an odd degree. We will proceed as follows:1) We employ the Horner’s method to verify the roots −1 ;1. Number 1 will be

certainly one of the roots of the equation.2 5 0 -3 3 0 -5 -2

1 2 7 7 4 7 7 2 0-1 2 5 2 2 5 2 0-1 2 3 -1 3 2 0

We can see that 1 is a simple root and −1 is a double root of the equation. If these numbers were to be tested further on, we would not get zero in the last column of the Horner schema anymore, as you can check by yourselves. Hence we can rewrite the equation into the form

( x−1 ) ¿

and we can notice the expression in the third parentheses form a reciprocal equation again, this time of the 1st class and of an even degree.

Page 47: Polynomials - Pedagogická fakulta MU€¦ · Web viewFrom the algebraic point of view it is especially interesting that the result of adding and multiplying polynomials is again

2) We rearrange the equation 2 x4+3 x3−x2+3 x+2=0 by dividing it by x2, writing the terms with the same coefficients together:

2(x2+ 1x2 )+3( x+ 1

x )−1=0

3) Before we perform the substitution x+ 1x=t , there needs to be determined how to

express x2+ 1

x2 :

(x+ 1x )

2

=t2 x2+2+ 1x2 =t2 x2+ 1

x2=t 2−2

When substituting the obtained expressions into the equation

2(x2+ 1x2 )+3( x+ 1

x )−1=0, we get

2 (t 2−2 )+3 t−1=0 2 t2+3 t−5=0 t1=−52

, t2=1.

Let us return to the original variable. When ¿−52 , then x+ 1

x=−5

2 , we multiply this

equation with 2 x (assuming that x≠ 0¿ and get the equation 2 x2+5 x+2=0, whose

solutions are the numbers x1=−2 , x2=−12 .

When t=1, then x+ 1x=1 and with a rearrangement similar to the previous step

we get the equation x2−1+x=0, whose solutions are x3,4=1 ±i √3

2.

The complete solution is represented by the set

{−2 ,−1 ,−12

,1 , 1−i √32

, 1+i √32 }.

Example 42 Solve individually the reciprocal equation 3 x8+ x7−4 x6−x5+2 x4−x3−4 x2+x+3=0.

Result:

x∈ {−1,1, 1± i√32

,−2±i √53 }.

47

Page 48: Polynomials - Pedagogická fakulta MU€¦ · Web viewFrom the algebraic point of view it is especially interesting that the result of adding and multiplying polynomials is again

Exercises for Check

1. Define algebraic equation and its roots.2. State and prove Bézout's theorem.3. Is the field of real numbers algebraically closed? Explain.4. Write down Vieta’s formulas for a polynomial of the 5th degree.

5. Find out multiplicity of the roots c1=12

, c2=−2 for the polynomial

f ( x )=8 x7+4 x6−2 x5+3 x4−6 x3−2x2+4 x−1.

6. Apply a derivative to find out multiple roots of the polynomial f ( x )=4 x5+20 x4+25 x3−10 x2−20 x+8∈Q [ x ].

7. Given the quadratic equation x2−5 x+3=0, where c1 , c2 are its roots, determine the value of the expression (c1+c2 )2−3c1 c2.

8. On the example of the polynomial f ( x )=4 x4−4 x3+13 x2−12 x+2 explain how rational roots can be found for a polynomial with integer coefficients. Find all rational roots.

9. Solve the inequality 4 x4−4 x3+13x2−12 x+2<0 on the set of real numbers.

10. On the set of complex numbers solve the equality 5 x4−26 x3+10 x2−26 x+5=0.

Results:

5. c1 is a triple root, c2 is not a root.

6. c1=12 is a triple root, c2=−2 is a double root.

7. 16

8. c=12 .

9. No solution.

10. x∈ {i ;−i ;5 ; 15 }.

Page 49: Polynomials - Pedagogická fakulta MU€¦ · Web viewFrom the algebraic point of view it is especially interesting that the result of adding and multiplying polynomials is again

4 Multivariate Polynomials

Chapter Objectives

Having studied this chapter you will be able to: define multivariate polynomial determine the degree of a multivariate polynomial lexicographically order a multivariate polynomial define symmetric and homogeneous polynomial define elementary symmetric polynomials.

Time to Study

Circa 2 hours

Study Guide

Definition 18 A polynomial of n variables x1 ,…, xn stands for a sum of a finite number of terms

f =A1+ A2+…+ A r ,

where Ai=ai x1k1 x2

k2… xnkn, a i∈T is the coefficient of the term Ai, and k1 ,…, kn are non-

negative integers. A polynomial of n variables over a field T is labelled with the symbol f ∈T [x1 ,... , xn]. The degree of a term x1

k1 x2k2 … xn

kn stands for the number k1+k2+…+kn.

Example 43 The degree of the term 5 x3 y5 is 8.

Note: In the following text the notion of term height will also come handy. Let f (x1, …, xn) be a polynomial of n variables and a i x1

k1 x2k2 … xn

kn be its arbitrary term. Then the sorted n-tuple of non-negative integers (k¿¿1 ,…,kn)¿ is called a height of the term.

Definition 19 The degree of a non-zero polynomial f ∈T [x1 ,... , xn] is equal to the highest of the degrees of its terms with non-zero coefficients. A polynomial whose all terms have the same degree s is called a homogenous polynomial (of the degree s).

49

Page 50: Polynomials - Pedagogická fakulta MU€¦ · Web viewFrom the algebraic point of view it is especially interesting that the result of adding and multiplying polynomials is again

Example 44 The polynomial x+ y is a homogenous polynomial of the degree 1, the polynomial x2−2 xy is a homogenous polynomial of the degree 2, while the polynomial x2 y+3 y3 is a homogenous polynomial of the degree 3.

Definition 20 Terms A=a x1k1 x2

k2 …xnkn, B=b x1

l1 x2l2 … xn

ln are called similar terms if and only if k1=l1 , k2=l2 , …,kn=ln.

Note: When possible we sum up all similar terms outright. There applies A+B=(a+b)x1

k1 x2k2 … xn

k n.

Definition 21 Considering operations +, ∙ , the set T [ x1 ,…,xn ] is an integral domain (operations are defined generally, ¿ is a group and (T [ x1 , …, xn ] ,∙) is a commutative semigroup).

Theorem 26 Every polynomial f ∈T [x1 ,... , xn] can be written in the form of a sum of homogenous polynomials, each of them of a different degree. This expression is unambiguous (except for their order).

Note: In case of univariate polynomials the terms can be ordered in the ascending or descending manner with respect to the variable exponent. Therefore some other procedure must be defined.

Theorem 27 Let A=x1k1 x2

k2 …xnkn, B=x1

l1 x2l2 … xn

ln be two terms of n variables. Let us state that the term A comes before the term B if there exists an index i ,1 in, for which there applies

k1=l1 , ... , k i 1=li1 , k i li .

The condition that the term A comes before the term B, or A=B, is denoted as A≫B.

Theorem 28 The relation ≫ is a relation of a linear ordering over the set of all terms containing n variables (the relation is irreflexive, antisymmetric and transitive).

Definition 22 The relation ≫ is called a relation of lexicographical order of terms of n variables. When the terms of a polynomial f T [ x1 , ... , xn] are ordered by this relation, we can state that the terms of the polynomial f are lexicographically ordered. The term that comes before all other terms of the polynomial f is called a leading term of the polynomial f .

Page 51: Polynomials - Pedagogická fakulta MU€¦ · Web viewFrom the algebraic point of view it is especially interesting that the result of adding and multiplying polynomials is again

Example 45 Determine a lexicographical order of the polynomial f ( x , y , z )=( x−2 y2 )(x2 z+z ) and its leading term.

Solution: Upon factorization we get f ( x , y , z )=x3 z+xz−2x2 y2 z−2 y2 z=x3 z−2x2 y2 z+xz−2 y2 z, which is the lexicographical order of the polynomial. We can verify it using the term heights: (3,0,1 ) , (2,2,1 ) , (1,0,1 ) ,(0,2,1) . The leading term of the polynomial is x3 z.

Theorem 29 Let f , g∈T [ x1 , ... , xn] be two arbitrary real non-zero polynomials of n variables. Then the product of the leading terms of the polynomials f and g is the leading term of the product f . g.

Definition 23 A polynomial f (x1 , ... , xn)∈T [ x1 , ... , xn] is called to be symmetric, when it does not change after any permutation of variables, i.e. for any arbitrary permutation (❑1 , ...,❑n) of indices 1 ,2 , ... , n there applies:

f ( xα1, …, xα n )=f (x1 , ..., xn)

The set of all symmetric polynomials of n variables over the field of real numbers is denoted as R s[x1 ,... , xn].

Note: A symmetric polynomial of two variables fulfils the condition f ( x , y )=f ( y , x ), whereas a symmetric polynomial of three variables fulfils the condition f ( x , y , z )=f ( x , z , y )=f ( y , x , z )=f ( y , z , x )=f ( z , x , y )=f (z , y , x). The polynomial x+ y is a symmetric polynomial of the degree 1, the polynomial x2+ x+ y+ y2 is a symmetric polynomial of the degree 2.

Theorem 30 Let A=x1k1 x2

k 2 …xnkn be the leading term of a symmetric polynomial

f (x1 , ... , xn). Then there applies k1 k2 ... kn.

Example 46 Find out a lexicographical order of the symmetric polynomial f ( x , y , z )=( x+ y ) ( x+z )( y+z ) and its leading term.

Solution: We proceed by factorization, getting f ( x , y , z )=x2 y+x2 z+x y2+2xyz+x z2+ y2 z+ y z2, which is the lexicographic order of the polynomial. The leading term of the polynomial is x2 y.

Theorem 31 Let A=x1k1 x2

k2 …xnkn be a term consisting of n variables. Then there

exists only a finite number of leading terms of symmetrical polynomials of n variables that come after the term A.

51

Page 52: Polynomials - Pedagogická fakulta MU€¦ · Web viewFrom the algebraic point of view it is especially interesting that the result of adding and multiplying polynomials is again

Example 47 Given the polynomial f ( x , y , z )=x3 y+2 x2 y2+x3 z+2 x2 z2−x2 y2 z2+x y3+x z3+2 y2 z2+ y3 z+ y z3, determine all its leading terms.

Solution: The heights of the terms of the polynomial are (3,1,0 ) , (2,2,0 ) , (3,0,1 ) , (2,0,2 ) , (2,2,2 ) , (1,3,0 ) , (1,0,3 ) , (0,2,2 ) , (0,3,1 ) , (0,1,3 ) respectively. When we order these heights by the relation ¿ we get (3,1,0 )>(3,0,1 )>(2,2,2 )>(2,2,0 )>(2,0,2 )> (1,3,0 )> (1,0,3 )> (0,3,1 )>(0,2,2 )>(0,1,3 ). The leading term of f is thus the term x3 y . However, the inequalities k1 k2 ...kn from Theorem 30 apply also for heights (2,2,2) and (2,2,0), and hence there are other leading terms of the polynomial x2 y2 z2 and 2 x2 y2.

We could conclude that the given polynomial is the sum of three elementary symmetric polynomials, whose leading terms are x3 y , x2 y2 z2, 2 x2 y2. The procedure for the decomposition of symmetric polynomials into a sum of elementary symmetric polynomials will be presented in the following chapter.

4.1 Elementary Symmetric Polynomials

Definition 24 Polynomials from R[ x1 , ... , xn] in the form:

σ 1(x1 , …, xn)=x1+ x2+…+ xn

σ 2( x1 , …, xn)=x1 x2+x1 x3+…+xn−1 xn

σ n−1(x1 , …, xn)=x1 x2 …xn−1+x1 x2 …xn−2 xn+…+x2 x3… xn−1 xn

σ n(x1 , …, xn)=x1 x2 x3 … xn−1 xn

are called elementary symmetric polynomials of n variables.

Note: The elementary polynomial ❑1 is therefore the sum of all variables x1 , ... , xn, the polynomial ❑2 is the sum of products of all pairs, the polynomial ❑3 is the sum of triples, and so forth. Finally, the polynomial ❑n is the product of all variables x1, ... , xn. Considering the case n=3 and the familiar variables notation x , y , z then:

❑1=x+ y+ z

❑2=xy+xz+ yz

❑3=xyz

Symmetric polynomials appear in Vieta’s formulas and in addition they can be used to solve some systems of equations which it is not possible to solve by methods known from elementary and high schools, as will be demonstrated.

Page 53: Polynomials - Pedagogická fakulta MU€¦ · Web viewFrom the algebraic point of view it is especially interesting that the result of adding and multiplying polynomials is again

The following is the fundamental theorem of symmetric polynomials which deals with the existence and unambiguity of an arbitrary symmetric polynomial expression, using elementary symmetric polynomials.

Theorem 32 Every symmetric polynomial f (x1 , ... , xn)∈RS[ x1 , ... , xn] can be expressed as a polynomial of n variables ❑1,❑2 ,... ,❑n over R, i.e.

f ( x1 ,... , xn )=φ(❑1 ,❑2 , ... ,❑n)

and this expression is unambiguous.

Example 48 Concerning power sums sk=x1k+x2

k upon application of the binomial theorem

(a+b )n=(n0)an+(n1)an−1b+…+( nn−1)a bn−1+(nn)bn

we gets1=x1+x2=σ1,s2=x1

2+x22=( x1+ x2 )2−2 x1 x2=σ1

2−2 σ2,

s3=x13+x2

3=( x1+x2)3−3 x1 x2(x1+x2)=σ13−3 σ1 σ2,

and so on. The following algorithm can be generically applied to the transformation of a symmetric polynomial into the sum of elementary symmetric polynomials.

Algorithm for Determining All Leading Terms of Symmetric Polynomials of the

Degree k1+k2+…+kn, That Come After The Leading Term ax1k1 x2

k2 … xnkn (k1 k2 ...kn):

1. Write down the height of the leading term of the polynomial, i.e. the sequence of exponents k1 , …, kn.

2. Find in this sequence the last number greater than 1. Decrement this number by 1, leave the fore-going numbers as they are and select the numbers that go “after” as small as possible for it to apply l1l2 ...ls and l1+l2+…+ls=k1+k2+…+kn.

3. Repeat the procedure until obtaining the sequence 1, 1, …, 1.Then the obtained sequence of the term heights describes all leading terms of all symmetric polynomials of the given order that come after the given leading term.

When seeking the symmetric polynomial f ( x1 ,... , xn ) in the form φ (❑1 ,❑2 , ...,❑n), to the first leading term a x1

k1 x2k2 … xn

kn we will create the corresponding term φ1=σ1

k1−k2 ∙ σ2k2−k3 ∙…∙ σn

kn. The similar process is then used for other leading terms.

The polynomial f can be written in the form f =a φ1+ A φ2+B φ3+…+W φ t, where a is the coefficient of the first leading term and the constants A , B , … are undetermined coefficients that can be found by substitution of concrete values for x1 , …, xn.

53

Page 54: Polynomials - Pedagogická fakulta MU€¦ · Web viewFrom the algebraic point of view it is especially interesting that the result of adding and multiplying polynomials is again

Example 49 Transform the polynomial f =x12 x2

2+x12 x3

2+x12 x4

2+x22 x3

2+… into the form φ (❑1 ,❑2 , ...,❑n).

Solution: Firstly, we set up a sequence of exponents and create corresponding leading terms of a general symmetric polynomial whose leading term is x1

2 x22. Such polynomial

should be of the form: x12 x2

2+x12 x3

2+ x12 x4

2+…+x12 x2 x3+x1

2 x2 x4+…+x1 x2 x3 x4. For each leading term we then create a corresponding term φk:

2 2 φ1=σ12−2 σ2

2=σ22

2 1 1 φ2=σ12−1 σ2

1−1 σ31=σ1σ 3

1 1 1 1 φ3=σ11−1 σ2

1−1 σ31−1σ 4

1=σ4

Therefore, for the polynomial f there applies

f =a σ22+ A σ1 σ3+B σ4 ,

where a=1 and the coefficients A , B will be computed by two choices of substitution for x1 , x2 , x3 , x4.

Case 1: ( x1 , x2 , x3 , x4 )=(1,1,1,0), f =x12 x2

2+…=1+1+1=3, σ 1=x1+x2+x3=1+1+1=3, σ 2=x1 x2+x1 x3+x2 x3=1+1+1=3, σ 3=x1 x2 x3=1, σ 4=0. After substitution into the equation f =σ 2

2+ A σ1 σ3+B σ4 we get

3=9+3 A A=−2.

Case 2: ( x1 , x2 , x3 , x4 )=(1,1,1,1), you can perform the substitution by yourselves to check that f =6 , σ 1=4 , σ2=6 , σ3=4 , σ4=1. Hence there applies

6=36+16 A+B B=2.This leads us to the sought form of the polynomial

f =σ 22−2 σ1 σ3+2 σ4 .

Example 50 Transform the symmetric polynomial f =( x− y )2 ( x−z )2 ( y−z )2 into the variables ❑1 ,❑2 ,... ,❑n.

Solution: Upon factorization we would get f =x4 y2−2 x4 yz+x 4 z2−2 x3 y3+… Thus we can write down the sequence of exponents of leading terms, striking out the rows where the sequence has more than 3 terms, since the polynomial f has only 3 variables.

4 2 φ1=σ12 σ2

2

4 1 1 φ2=σ13 σ3

3 3 φ3=σ23

Page 55: Polynomials - Pedagogická fakulta MU€¦ · Web viewFrom the algebraic point of view it is especially interesting that the result of adding and multiplying polynomials is again

3 2 1 φ4=σ1 σ2 σ3

3 1 1 1

2 2 2 φ6=σ 32

2 2 1 1

2 1 1 1 1

1 1 1 1 1 1

f =σ 12σ 2

2+ A σ13 σ3+B σ2

3+C σ1 σ2 σ3+D σ 32 .

We have 4 undetermined coefficients, therefore 4 different choices will have to be performed (notice that for instance the cases (1,1,0) a (0,1,1) are equivalent with respect to the result, which is caused by the symmetrisation of the polynomial):

Case 1: ( x , y , z )=(1,1,0), f =( x− y )2 ( x−z )2 ( y−z )2=0 , σ1=x+ y+z=2 , σ2=xy+xz+ yz=1, σ 3=xyz=0

0=4+B B=−4

Case 2: ( x , y , z )=(1,1,1), f =0 , σ1=3 , σ2=3 , σ3=1

0=81+27 A−27.4+9 C+ D27=27 A+9C+D

Case 3: ( x , y , z )=(1,1 ,−2), f =0 , σ1=0 , σ2=1.1+1. (−2 )+1. (−2 )=−3 , σ3=−2

0=27.4+4 D D=−27

Case 4: ( x , y , z )=(1,1 ,−1), f =0 , σ1=1, σ 2=1.1+1. (−1 )+1. (−1 )=−1, σ 3=−1

0=1−A+4+C−27 22=−A+CNow we will solve the following system of equations for the remaining undetermined coefficients:

27=27 A+9C−2722=−A+C

The solution of this system is A=−4 ,C=18. To summarise

f =σ 12σ 2

2−4 σ13 σ3−4σ 2

3+18 σ1 σ2 σ3−27 σ 32 .

Example 51 Solve individually: Transform the polynomial f =x3+ y3+ z3 into the variables ❑1 ,❑2 ,... ,❑n.

Result: f =σ 13−3 σ1 σ2+3 σ3

Example 52 Solve individually: Transform the polynomial f =( x2+ y2 ) ( x2+z2 )( y2+z2) into the variables ❑1 ,❑2 ,... ,❑n.

Result: The leading term x4 y2, f =σ 12σ 2

2−2 σ13 σ3−2 σ 2

3+4 σ 1σ 2σ 3−σ32

55

Page 56: Polynomials - Pedagogická fakulta MU€¦ · Web viewFrom the algebraic point of view it is especially interesting that the result of adding and multiplying polynomials is again

Symmetric polynomials have their use in application tasks. However, only two simple examples will be shown here to illustrate the usefulness of symmetric polynomials in various types of tasks.

Example 53 Solve the following system of equations in the domain of real numbersx3+ y3=35xy=6.

Solution: We can compute and check that x3+ y3=σ 13−3 σ1 σ2 and xy=σ2. Thus we get a

new system of equations

σ 13−3 σ1 σ2=35

σ 2=6

After substitution of the second equation into the first one, we get the equation σ 1

3−18σ1−35=0, by estimating and substituting into the Horner scheme stepwise we eventually find one real root σ 1=5. As σ 1=x+ y, we will now be solving the system

x+ y=5xy=6 ,

whose solution is the set of two ordered pairs { [ 3,2 ] , [ 2,3 ]}.We can perform the check by substituting into the original assignment and having

done this in both cases, we come to the identity 35=35 6=6.

Example 54 A cuboid has edges of lengths a , b , c. There applies a+b+c=6 and ab+ac+bc=9. Determine the volume of the cube that has the body diagonal of the same length as the given cuboid.

Solution: Let us mark the cube edge length with k . The body diagonal of the cuboid is of the length √a2+b2+c2 while the length of the cube body diagonal is √3 k. This leads to the equation

√a2+b2+c2=√3 k ,which after exponentiation turns into

a2+b2+c2=3 k2 .

The polynomial a2+b2+c2 is a symmetric polynomial and there applies a2+b2+c2=σ12−2 σ2

, hence

σ 12−2 σ2=3 k 2 .

At the same time we know from the assignment that σ 1=a+b+c=6 and σ 2=ab+ac+bc=9. Thus

62−2.9=3 k 2 ,

Page 57: Polynomials - Pedagogická fakulta MU€¦ · Web viewFrom the algebraic point of view it is especially interesting that the result of adding and multiplying polynomials is again

After rearrangement k 2=6, k=√6. The sought volume of the cube is then

V=k3=6 √6 .

Example 55 Solve the following system of equations individually:x3+ y3=28x+ y=4

Solution: { [ 3,1 ] , [ 1,3 ] }.

57

Page 58: Polynomials - Pedagogická fakulta MU€¦ · Web viewFrom the algebraic point of view it is especially interesting that the result of adding and multiplying polynomials is again

Exercises for Check

1. Define polynomial of n variables and its degrees.2. Define homogeneous and symmetric polynomial.3. Determine the degrees of the following polynomials. Are they homogeneous or

symmetric?a. x− y ,b. 3 xy+2 x2,c. 4 x y2+3 x+3 y+4 x2 y,d. 5 x+ y2−7,e. x y2+ y z2+z x2,f. x y2+ y z2+z x2+x2 y+ y2 z+z2 x.

4. Apply the binomial theorem to rewrite the sum of powers s4=x14+x2

4 by elementary symmetric polynomials.

5. Express the polynomial ( x+ y ) ( y+z )(z+x ) using elementary symmetric polynomials.

Results:

3. a. 1, homogeneous, b. 2, homogeneous, c. 3, symmetric, d. 2, neither homogeneous nor symmetric,, e. 3, homogeneous, f. 3, homogeneous and symmetric

4. s4=σ14−4 σ1

2σ 2+2 σ 22

5. σ 1σ 2−σ3.

Page 59: Polynomials - Pedagogická fakulta MU€¦ · Web viewFrom the algebraic point of view it is especially interesting that the result of adding and multiplying polynomials is again

References

BARBEAU, E. J.: Polynomials. Springer Verlag New York Inc., 1989

BIRKHOFF, G., MAC LANE, S.: Prehľad modernej algebry. Prague: SNTL, 1979

BLAŽEK, J., KOMAN, M., VOJTÁŠKOVÁ, B.: Algebra a teoretická aritmetika. Prague: SPN, 1985

HERMAN, J., KUČERA, R., ŠIMŠA, J.: Metody řešení matematických úloh I. Prague: PřF MU, 1996

HORÁK, P.: Polynomy. 1. vydání. Brno: UJEP, 1978.

59